AR exam 2

¡Supera tus tareas y exámenes ahora con Quizwiz!

169. Limited "floor" would be the biggest problem when a test will be used to A. distinguish between mildly and moderately retarded children. B. distinguish between above-average and gifted students. C. distinguish between successful and unsuccessful trainees. D. distinguish between satisfied and dissatisfied customers.

A ("Floor" refers to a test's ability to distinguish between examinees at the low end of the distribution, which would be an issue when distinguishing between those with mild versus moderate retardation. Limited floor occurs when the test does not contain enough easy items. Note that "ceiling" would be of concern for tests designed to distinguish between examinees at the high end of the distribution -answer B-.)

195. The "terminal drop" phenomenon refers to: A. declines in intellectual functioning B. declines in sensory functioning C. declines in REM sleep D. declines in non-REM sleep

A ("Terminal drop" refers to findings indicating in the months before death, a substantial drop in all facets of intelligence occurs.)

117. All of the following disorders occur more frequently in children with Tourette's Disorder than in the general population except A. Generalized Anxiety Disorder. B. Obsessive-Compulsive Disorder. C. Attention Deficit/Hyperactivity Disorder. D. Learning Disorders.

A (According to the DSM-IV, Obsessive-Compulsive Disorder, Attention-Deficit/Hyperactivity Disorder and Learning Disorders are all associated more with children with Tourette's Disorder. However, Generalized Anxiety Disorder is not identified as such.)

106. A diagnosis of Mental Retardation is dependent upon cognitive functioning of two standard deviations below the mean and A. deficits in adaptive functioning. B. lack of social skills. C. cognitive deficits in at least two academic areas. D. an inability to inhibit impulses.

A (According to the DSM-IV, the three criteria for a diagnosis of Mental Retardation are -1- significantly subaverage intellectual functioning -an IQ score of 70 or less-, -2- deficits in at least two areas of adaptive functioning, and -3- onset before age 18. The other choices may be part of the clinical picture of Mental Retardation, but they are not diagnostic criteria.)

186. The effectiveness of Eye Movement Desensitization and Reprocessing -EMDR- in the treatment of Posttraumatic Stress Disorder is due to: A. exposure to anxiety-provoking stimuli B. changes in the functioning of the brain due to lateral eye movement stimulation C. unconditional positive regard D. placebo effect

A (Eye Movement Desensitization and Reprocessing -EMDR- is a controversial therapeutic technique which combines techniques from cognitive-behavioral and other treatment approaches with lateral eye movements. Clients receiving EMDR are asked to recall anxiety-provoking memories and are then instructed to track the therapists' fingers which are moved from side to side. This process is repeated until the clients' anxiety is extinguished. Although Francine Shapiro, the developer of the technique, originally proposed that the bilateral eye movements accelerate the reprocessing of traumatic memories, a meta-analysis of 34 studies indicated that the eye movement component did not contribute to the effectiveness of EMDR. Rather, it appears that the element of exposure is responsible for its effects. The meta-analysis did find EMDR to be more effective than no-treatment or non-exposure treatments; however, it was no more effective than other exposure techniques. -P. R. Davidson, & K. C. Parker, Eye movement desensitization and reprocessing -EMDR-: A meta-analysis. Journal of Consulting and Clinical Psychology, 2001, 69-2-, 305-316-.)

165. According to Vygotsky, what has the greatest impact on the development of language? A. social relationships B. internal cognitive structures C. schemas D. scaffoldings

A (For Vygotsky, language is originally and primarily social. Thus a child's language and cognitive processes are greatly influenced by his social relationships and culture. From these experiences the child formulates his language. This is in contrast to Piaget -answer B and C- who believed that universal internal cognitive structures or schema developed first within the child, and then were influenced by his surroundings. Answer D "scaffolding" is Vygotsky's term for the assistance adults give to children to help them learn about the world.)

137. Of the following ethnic groups which group is not considered a race? A. Hispanic B. Native American C. African-American D. Asian-American

A (Hispanics are an ethnic group like African-Americans, Asian-Americans, and Native Americans. Unlike these groups, Hispanics are not a specific race. According to the 1998 Census Bureau, the total Hispanic population of the US accounts for 11% of the total population. Sixty-three percent are of Mexican origin, 14.4% are of Central and South American origin, 10.6% of mainland Puerto Rican origin, 4.2% of Cuban origin, and 7.4% other -Alfredo Ardilis, Testing Hispanic Populations, Texas Psychologist, Winter 2000)

193. An employer is concerned about the high voluntary turnover rates in her company. Which of the following would be most useful for ensuring that she identifies job applicants who are at high-risk for quitting their jobs if they are hired? A. a biographical information form B. an interview C. a personality test D. a measure of "g"

A (Meta-analyses of the research on the validity of various selection techniques has looked at the relationship between different predictors and different outcome measures. Of the various commonly-used predictors, biographical information has the highest correlation with turnover.)

132. A company wants its clerical employees to be very efficient, accurate and fast. Examinees are given a perceptual speed test on which they indicate whether two names are exactly identical or slightly different. The reliability of the test would be best assessed by: A. test-retest B. Cronbach's coefficient alpha C. split-half D. Kuder-Richardson Formula 20

A (Perceptual speed tests are highly speeded and are comprised of very easy items that every examinee, it is assumed, could answer correctly with unlimited time. The best way to estimate the reliability of speed tests is to administer separately timed forms and correlate these, therefore using a test-retest or alternate forms coefficient would be the best way to assess the reliability of the test in this question. The other response choices are all methods for assessing internal consistency reliability. These are useful when a test is designed to measure a single characteristic, when the characteristic measured by the test fluctuates over time, or when scores are likely to be affected by repeated exposure to the test. However, they are not appropriate for assessing the reliability of speed tests because they tend to produce spuriously high coefficients. )

164. In principal components analysis, an eigenvalue would indicate A. the amount of variability in a group of variables accounted for by an independent statistical component. B. the amount of variability in a group of variables accounted for by a statistical component that shares variability with other statistical components in the analysis. C. the amount of variability in one measured variable accounted for by all the independent statistical components in the analysis. D. the amount of variability in all measured variables accounted for by all the statistical components in the analysis.

A (Principal components analysis and factor analysis are two complex statistical techniques designed to determine the degree to which a large set of variables can be accounted for by fewer, underlying constructs -referred to as "factors" or "principal components"-. In principal components analysis and factor analysis, an eigenvalue is a statistic that indicates the degree to which a particular factor is accounting for variability in the variables studied. In other words, a factor's eigenvalue indicates its strength or explanatory power. The reason choice "A rather than "B is correct is that in principal components analysis, the factors or components are always independent, or uncorrelated.)

161. An adolescent's behavior is motivated primarily by her needs for power and attention. Her problems would probably be best addressed by a therapist adopting the approach of: A. Adler. B. Ellis. C. Perls. D. Freud.

A (Probably the best way to approach this question is to see if the notions of power and attention are linked to any of the people listed in the responses. For Adlerians, a desire to belong is a primary motivator of behavior, but this desire may be channeled into the mistaken goals of power, attention, inadequacy, or revenge.)

218. A therapist using Beck's cognitive approach to therapy would rely primarily on which of the following to induce desirable changes in a depressed client? A. Socratic questioning B. interpretation C. reflecting D. clarification and explanation

A (Questioning is a very important strategy in Beck's cognitive therapy and, in fact, the majority of communications by the therapist take the form of questions designed to help the client consider particular issues, options, and so on.)

124. A primary tenet of rational-emotive therapy is that: A. a belief determines behavior. B. self-attributions determine behavior. C. conditioning determines behavior. D. internalized role constructs determine behavior.

A (RET is basically a cognitively-based theory even though in practice therapists use many modalities. The idea is that beliefs -- irrational beliefs -- determine our maladaptive behaviors. The belief that you mustn't make anyone mad at you is an example.)

107. Severing the corpus callosum will have the greatest impact on which of the following? A. sensory systems B. motor systems C. memory D. mood

A (Recalling the studies on split-brain patients would have helped you answer this question. Remember that these patients had trouble verbally identifying stimuli received by sensory systems on the left side because those signals were received by the right hemisphere and not "shared with" the left hemisphere.)

82. Premature termination would be most likely to occur when A. an African-American client has a Caucasian therapist. B. an African-American client has an African-American therapist. C. a Caucasian client has an African-American therapist. D. a Caucasian client has a Caucasian therapist.

A (Research with African-American clients on the relationship between therapist-client racial similarity and therapy effectiveness has yielded contradictory results and suggests that this relationship is mitigated by a variety of factors. A clearer relationship exists between similarity and premature termination, with the probability of premature dropout increasing when an African-American client works with a racially dissimilar therapist.)

149. The "slippery slope" argument suggests therapist-patient sexual relationships result from: A. nonsexual boundary crossings and violations B. confusion about the definition of dual relationships and how to address them C. personal characteristics or life circumstances of the therapist D. previous sexual relation with therapist, supervisor or educator

A (The "slippery slope" analogy refers to the relationship between sexual and non-sexual boundary-crossing and claims that crossing one boundary makes it easier to cross the next boundary. Research has implicated three factors as influences on the occurrence and continuation of exploitative sexual and nonsexual relationships with clients, supervisees, and students: the connection between sexual and nonsexual professional boundary crossings or violations; the confusion about the definition of dual relationships and how to address them; and personal or contextual characteristics that put psychologists at risk for engaging in such behavior. Research has not found a significant connection between reported sexual boundary violations as a former client, supervisee, or student and as a practicing professional -d.-. -See: Lamb, D., Catanzaro, S., & Moorman, A. -2003-. Psychologists reflect on their sexual relationships with clients, supervisees, and students: Occurrence, impact, rationales, and collegial intervention. Professional Psychology: Research and Practice, 34, 102-107; Williams, M. -1997-. Boundary violations: Do some contended standards of care fail to encompass commonplace procedures of humanistic, behavioral, and eclectic psychotherapies? Psychotherapy, 34, 238-249; and Pope, K. -1990-. Therapist-patient sexual involvement: A review of the research. Clinical Psychology Review, 10, 477-490.)

126. Negative reinforcement is essential to: A. escape conditioning. B. punishment. C. aversion conditioning. D. phobias.

A (The concept of negative reinforcement involves increasing the likelihood of a behavior through the elimination of an aversive stimulus -the negative part-. As you might recall, this is exactly what the dog does by learning to escape the electric shock by jumping to the other side of the cage. This is the classic example of escape conditioning. Jumping to the other side is reinforced by the elimination of the aversive stimulus. You may have chosen punishment, but this is the common error of confusing negative reinforcement with punishment. To avoid this error, keep in mind that reinforcement -whether negative or not- and punishment are antithetical.)

198. Which of the following represents a good reason to use interval recording in the process of behavioral observation? A. the behavior being measured has no clear beginning or end B. the behavior has a sudden and discrete onset C. the behavior does not occur very often D. the behavior is poorly defined and it is therefore difficult to obtain agreement as to whether or not it is occurring

A (The interval recording method involves observing a behavior for a single block of time, such as 30-60 minutes per day. The block of time is divided into smaller intervals -e.g., 30 seconds-, and the behavior is recorded as having occurred or not occurred during each interval. This is a good method of recording behaviors with no fixed beginning or end, since the observer does not have to identify at what point the behavior began or ended -- he or she simply has to note whether it is occurring at all during a particular time interval.)

206. Among white males, the highest suicide rates are among those aged 75 and older. Among African-American males, the rates are highest for those: A. 25 to 44. B. 45 to 64. C. 65 to 74. D. 75 and above.

A (The suicide rates for whites and African-Americans have both increased in recent years. However, the highest rates for white males are among the elderly; for African-Americans, the highest rates are among those aged 25 to 44.)

139. Which of these studies would be considered most unethical today? A. Milgram's B. Bandura C. Zimbardo D. BF Skinner's study of his own child

A (This is a different twist on a previous question but the answer is still Milgram's study. You may feel a different study is more unethical, but the Ethical Standards are specifically pertinent to Milgram's study. This study involved deception and no debriefing about that deception. Ethical Standard 8.07 -Deception in Research- addresses this, indicating that: "-a- Psychologists do not conduct a study involving deception unless they have determined that the use of deceptive techniques is justified by the study's significant prospective scientific, educational, or applied value and that effective nondeceptive alternative procedures are not feasible. -b- Psychologists do not deceive prospective participants about research that is reasonably expected to cause physical pain or severe emotional distress. -c- Psychologists explain any deception that is an integral feature of the design and conduct of an experiment to participants as early as is feasible, preferably at the conclusion of their participation, but no later than at the conclusion of the data collection, and permit participants to withdraw their data.")

208. You are starting a stop-smoking group and tell a newspaper reporter who smokes that he can participate in the group for free if he'll write an article about it for the local newspaper. This arrangement is A. unethical. B. ethical as long as you don't tell the reporter what to write about. C. ethical as long as you check to make sure that information about the group is accurate. D. ethical as long as the confidentiality of the group participants is not violated.

A (This issue is addressed in Standard 5.02-b- of the Ethics Code, which states that "Psychologists do not compensate employees of press, radio, television, or other communication media in return for publicity in a news item." )

102. Bob, who came from a poor family, was encouraged throughout high school to take vocational classes due to his "low aptitude" scores. Bob ignored this advice and focused on college preparatory courses. After graduating Magna Cum Laude from Harvard Business School, he wrote his old alma mater and asked for a complete copy of his records. Bob's grandmother said, "While you are at it--tell them to burn mine". According to the Buckley Amendment, A. Overall, the school must comply with both requests B. They need to honor Bob's wishes but not his grandmother's C. They need to honor Bob's grandmother's request, but not Bob's D. Then can disregard both requests

A (This question is a little tricky. While they don't literally have to "burn" the records of Bob's grandmother, overall both requests should be honored. According to the Buckely Amendment or the Family Educational Rights and Privacy Act, eligible students after age eighteen and their parents have the right of access to their children's educational records and can challenge any content thereof. Also records that are no longer useful or relevant for the students or the educational institution are to be destroyed. Although Bob would not have any legal rights regarding the disposition of his grandmother's records, once the school is made aware of the obsolete records they should destroy them.)

192. The misinformation effect -hindsight bias- may be a form of: A. retroactive interference. B. motivated forgetting. C. repression. D. proactive interference.

A (To answer this question, you need to understand what retroactive and proactive interference are. Retroactive interference occurs when your ability to recall X is difficult because of interference by something you learned after X. The longer the period of time between learning X and being tested on it, the greater the opportunity for retroactive interference. Proactive interference occurs when the ability to recall X is impaired by previously learned material. Proactive interference can occur regardless of how long the interval is between learning X and recalling it. Finally, retroactive and proactive interference are most likely to be a problem for information that is not inherently meaningful, which would be the case for a set of unrelated words. )

213. Which of the following is not a factor related to outcome for individuals with autism? A. language ability B. age of onset C. level of intelligence D. severity of disorder

B (A diagnosis of Autism typically occurs between three to five years of age and symptoms of autism generally persist throughout life. In general, good outcomes have been characterized as the achievement of independence and a normal social life. The long-term outcome for individuals with autism is variable with only a small percentage living and working independently as adults. Prognosis findings have been consistent across a broad range of studies, with about 2% attaining normal functioning and approximately 40% high functioning. According to DSM-IV-TR, the best prognostic indicators have been found to be early language skills -presence or absence of speech-, and overall intellectual level. Another factor consistently related to outcome is severity of disorder. IQ alone best predicts only those with worst outcome. The prognosis is strongly influenced by how much usable language the child has acquired by age 7. Other correlated factors include: developmental milestones, social maturity and behavior, time in school and comorbid neuropsychiatric disorders. Factors unrelated to outcome include: birthweight, perinatal complications, age of onset, normal development before onset, SES, late development of seizures, type of treatment, family mental illness. )

66. Complex Partial Seizure Disorder, formerly known as Temporal Lobe Epilepsy, is known to originate in the temporal lobe. However, absence seizures, formerly known as petit mal seizures, are believed to originate in the: A. cerebelum B. thalamus C. occipital lobe D. parietal lobe

B (Absence or Petit Mal Seizures are very brief -30 seconds or less- and are characterized by minimal motor activity and a lack of awareness. Absence seizures usually begin in childhood and most outgrow the condition by adulthood. Researchers believe that absence seizures originate in the thalamus -the central relay station for sensory information going to the cortex-.)

190. From the perspective of Beck's cognitive-behavioral model, automatic thoughts are A. basic irrational beliefs that underlie depression and other maladaptive behaviors. B. interpretations of a situation that determine one's behavioral and emotional responses. C. central, firmly-held beliefs about the self. D. misinterpretations of situations that reflect some type of cognitive distortion.

B (According to Beck, everyone has automatic thoughts - they're not necessarily associated with cognitive distortions or psychological distress. However, when automatic thoughts provide an unrealistic interpretation of the situation -"This is awful!"-, they may lead to maladaptive behavioral or emotional responses.)

136. Hypnosis involves three factors A. Absorption, regression, dissociation B. Dissociation, absorption, suggestibility C. Suggestibility, dissociation, regression D. Regression, distortion, suggestibility

B (According to Hales, Yudofsky and Talbott, -1944-, hypnosis involves three factors, 1- absorption, whereby the individual is completely engrossed in a central experience, 2- dissociation, whereby the ordinary functioning of consciousness and memory are altered in some way and 3- suggestibility, whereby individuals have a tendency to be less inhibited and restricted while in the trance-like state.)

202. Which of the following is related to minority influence? A. Ambiguity B. Idiosyncracy credits C. Conformity D. Psychological reactance

B (According to Hollander -1985- in order to successfully challenge the majority opinions of a group, a person must first conform to the group in order to establish credibility as a competent insider. By becoming accepted members of the group, we accumulate idiosyncracy credits which are like brownie points.)

214. Two women golfers are competing in a golf tournament. Woman A is told by the coach to do her best. Woman B is told by her coach to try to shoot one under par on Holes 1, 4, and 7 and 11, and make par on all the rest. Which golfer is likely to do the best according to Locke? A. Golfer A B. Golfer B C. They'll do the same D. Golfer C, who will not be distracted from the game by verbal directions from a coach

B (According to Locke's 1970 goal-setting theory, goals serve two purposes-they are a basis for motivation and they direct behavior. Goal attainment is maximized when goals are specific and moderately difficult and when frequent feedback about progress toward goal achievement is provided. Locke is associated with Industrial Organizational Psychology, even though this question seems like it could be from the Social Psychology domain. Questions on the EPPP are often "disguiseD in this way.)

24. According to McClelland's Need for Achievement theory, individuals high in Need for Achievement will prefer: A. difficult tasks which bring very high acclaim when achieved. B. moderately difficult tasks which are both attainable and difficult enough to satisfy the person's need to feel a sense of accomplishment. C. easy tasks which are very easy to achieve and therefore provide immediate gratification of the person's needs. D. any task which gives the person the opportunity for recognition and control over others.

B (According to McClelland's theory, three needs underlie motivation: need for achievement, need for power, and need for affiliation. Individuals high in need for achievement tend to seek autonomy, personal responsibility, and recognition for their efforts. Since the need for achievement is balanced against a need to avoid failure, these individuals tend to prefer moderately difficult goals and tasks.)

41. Relapse by smokers who have stopped smoking is most likely to be triggered by the presence of smoking cues in one's surroundings or A. positive affect. B. negative affect. C. inactivity. D. coffee consumption.

B (According to S. Shiffman et al., the most common precipitants of relapse are the presence of smoking cues and negative affect -First lapses to smoking: Within-subjects analysis of real-time reports, Journal of Consulting and Clinical Psychology, 64-2-, 366-389-.)

114. Which of the following groups has the highest rate of completed suicide? A. Caucasian females B. Caucasian males C. African-American females D. African-American males

B (Across all age groups, Caucasian males have a higher rate of completed suicide that the other groups listed by the choices. However, the rate of suicide in African-American males is rising the fastest.)

101. According to Rutter, the risk patterns that are accurate predictors for child psychopathology are: A. Small family size, parental criminality, marital discord B. Severe marital discord, low socioeconomic status C. Large family size, high socioeconomic status, parental psychopathology D. Low socioeconomic status, small family size

B (Again, getting this question correct involves a willingness to read carefully through the answer choices. As you learned if you took Test 3, Rutter listed low socioeconomic status, severe marital discord, large family size, parental criminality, and placement of children outside the home as predictors of child psychopathology.)

19. Symptoms of separation anxiety usually begin to appear at approximately: A. 5 months B. 9 months C. 12 months D. 18 months

B (Although separation anxiety sometimes begins as early as 5 or 6 months, it typically has its onset at 8 or 9 months of age, rises dramatically until the age of 18 months, and then gradually falls off until it becomes negligible between the ages of 24-36 months. Prior to the onset of separation anxiety, babies do not typically protest when separated from their primary caretakers, even though a number of attachment behaviors develop during this time.)

158. A research study using an ABAB design involves the following elements in sequence: A. treatment, baseline, treatment, baseline B. baseline, treatment, baseline, treatment C. baseline, intervention in setting A, baseline, intervention in setting B D. baseline for group A, treatment for group A, baseline for group B, treatment for group B

B (An ABAB design is a type of single-subject design. It is also an example of a reversal design - a baseline measure of a behavior is obtained -the "A phase-, the behavior is again measured after a treatment is administered -the "B phase-, the treatment is removed or reversed and the behavior is again measured -the second "A-, and the behavior is again measured after the treatment is re-applied -the second "B-. Choices C and D are examples of multiple baseline designs. Multiple baseline designs do not involve withdrawal of treatment, but rather, apply the treatment to multiple settings -multiple baseline across settings- or to the same behavior of different subjects -multiple baseline across subjects-.)

181. Dementia due to Head Trauma A. is usually progressive in cases of moderate to severe trauma. B. is usually progressive only in cases of repeated head trauma. C. unlike other forms of Dementia, does not usually involve disturbances in executive functions. D. unlike other forms of Dementia, is associated more with deficits in executive functions than with memory impairment.

B (As described in DSM-IV, Dementia due to Head Trauma is usually nonprogressive when it is the result of a single head trauma. If it is progressive, this usually indicates a superimposed condition -e.g., hydrocephalus-.)

216. Crystallized functions are considered to be: A. dependent on cultural factors B. dependent on learning and education C. culture-free D. independent of learning

B (Crystallized Intelligence -Gc- is the breadth and depth of a person's acquired knowledge of a culture and the effective application of this knowledge. According to the APA Dictionary of Psychology, crystallized abilities are those abilities "such as vocabulary and cultural knowledge, that are a function of learning and experience in a given culture. Crystallized abilities are believed to depend on physiological condition somewhat less than do fluid abilities; thus they may be better sustained in old age. They are also believed by some to derive from fluid abilities." )

50. Typically, the onset of the SSRIs antidepressant effect begins within a few: A. months B. weeks C. days D. hours

B (Current clinical practice indicates that at least 2 to 3 weeks of continuous SSRI use are necessary for the antidepressant effects to manifest in patients. The results of a meta-analysis conducted by Posternak & Zimmerman -2005-, of 47 double-blind, placebo-controlled trials conducted between 1981 and 2000, found most of the antidepressant effects occurred during the first 2 weeks. Review of the studies, found only 13 cohorts -19.7%- from 8 studies showed a pattern where there was an increased antidepressant effect from weeks 3 to 6 compared with weeks 1 to 3. -See: Posternak, M.A. & Zimmerman, M. -2005-. Is there a delay in the antidepressant effect? A meta-analysis. Journal of Clinical Psychiatry. 66-2-; 148-158.- Additionally, research on the time course of improvement for seven different types of antidepressants, compared to placebo, indicates almost no differences between treatment modalities and found mean values around day 12-14 for improvement and day 18-20 for response. -See: Stassen, H., Angst, J., & Szegedi, A. -2003- How to assess the onset of antidepressant effect: Empirical data on 2788 patients. Pharmacopsychiatry 36:265)

179. Cultural differences and factors have an influence on the manifestation of client symptoms. If a psychologist assumes an Asian client's somatic complaints are really reflective of a mental disorder it is an example of: A. demand characteristics B. diagnostic overshadowing C. the fundamental attribution bias D. self-perception bias

B (Diagnostic overshadowing is a term to describe when a therapist's ability to recognize or consider other symptoms or conditions is "overshadoweD by one aspect of a client's symptoms or condition. Demand characteristics -a.- are unintentional cues in the experimental environment or manipulation that allow participants to guess the hypothesis and, as a result, participants may behave differently than they would under normal conditions. The fundamental attribution bias -c.- refers to the tendency to overestimate dispositional -personality- factors and underestimate situational factors in explaining the behavior of others. Self-perception bias -d.- refers to an individual inferring what their internal state is by perceiving how they are acting in a given situation.)

73. Linehan's dialectical behavior therapy shows promise in the treatment of borderline patients. It involves a combination of treatment modalities that include A. inpatient therapy. B. social skills training. C. family therapy. D. psychotropic medication.

B (Dialectical behavior therapy or DBT consists of outpatient individual therapy, social skills training and follow-up telephone consultations. It also discourages the expression of overt inappropriate affect. Its combination of social skills training -usually done in group therapy settings- and outpatient individual psychotherapy has been found to decrease hospitalization and suicide attempt rates -M. Linehan and C.A. Kehrer, Borderline Personality Disorder, in D.H. Barlow, Clinical Handbook of Psychological Disorders: A Step-by-Step Treatment Manual, New York, Guilford, 1993-.)

130. In late childhood and adolescence, sibling relationships tend to become egalitarian; however, during middle childhood they can best be described as: A. distant and distrustful B. conflicted and close C. abusive and uncaring D. dominant and submissive

B (During middle childhood sibling relationships are characterized by a combination of conflict and closeness. During this period siblings typically fight and experience friction, yet also report having greater warmth and companionship with each other.)

59. The ratio of Major Depressive Disorder in females to males is approximately A. 1:1. B. 2.5:1. C. 5:1. D. 1:2.

B (Estimates of the female to male Major Depression rate range from 2:1 to 3:1. Thus, 2.5:1 is the best answer to this question.)

188. A father brings his 10-year-old son to a psychologist for therapy due to symptoms of depression. The father states that the boy's symptoms began shortly after the parents separated several months ago. The boy is currently living with his father. The father further states that he only wants help for his son and does not want any report or testimony from the psychologist for the divorce proceedings. The psychologist should: A. have both parents sign an agreement that neither will require the psychologist to provide a report or testimony about the child's therapy B. contact the parents' attorneys to clarify the psychologist's role C. refuse to treat the child unless you can evaluate both parents D. provide therapy for the child but do not maintain any records that could be subpoenaed

B (Ethical Standard 10.02-b- states, "If it becomes apparent that psychologists may be called on to perform potentially conflicting roles -such as family therapist and then witness for one party in divorce proceedings-, psychologists take reasonable steps to clarify and modify, or withdraw from, roles appropriately." Although the patient's father has indicated that he does not want a report or testimony from the psychologist, it would be more prudent to clarify the psychologist's role with the attorneys of each of the parents. Having the parents sign an agreement as suggested in "A does imply a clarification of the psychologist's role, but it may not be legally binding or in the child's best interests and is not as good a choice as "B." Choice "C is incorrect because although it is necessary to evaluate all members of a family before making recommendations regarding child custody, in this case, it is therapy that is requested, not a custody evaluation. Finally, "D is incorrect because psychologists must maintain adequate records.)

30. Which of the following aspects of the Type A behavior pattern, is, according to research, most strongly connected to increased risk for coronary heart disease? A. job involvement B. anger and hostility C. need for achievement D. impatience

B (Friedman and Rosenman -1974- described the Type A personality as someone who is competitive, achievement oriented, highly involved in his work, active, aggressive, and urgent about time and meeting deadlines. Originally, it was hypothesized that the Type A behavior pattern -TABP- increases the risk of coronary heart disease -CHD-. However, recent research suggests that some but not all aspects of the TABP increase the risk of CHD. Specifically, hostility, aggression, and )

225. The results of the Robber's Cave experiment -Sherif et al., 1961- indicated that: A. emotional reactions are based upon cognitive interpretations of arousal B. superordinate goals reduce hostility between groups C. we prefer to be right rather than happy D. we tend to seek, interpret, and create information that verifies our existing beliefs

B (In Sherif's Robber's Cave Study, children in a summer camp were divided into two groups on an arbitrary basis and made to engage in competition with each other. This engendered intergroup hostility and dislike, which was found to be greatly reduced when a task requiring the two groups to cooperate -on superordinate goals- was introduced. Choice A represents Schachter's two-factor theory of emotion. Choice C is a conclusion of Self Verification Theory. And Choice D exemplifies the confirmatory bias.)

157. A test has a reliability coefficient of .90. What percentage of variability among examinees on this test is due to true score differences? A. 100% B. 90% C. 81% D. 50%

B (In most cases, you would square the correlation coefficient to obtain the answer to this question. However, the reliability coefficient is an exception to this rule: it is never squared. Instead, it is interpreted directly. This means that the value of the reliability coefficient itself indicates the proportion of variance in a test that reflects true variance.)

224. A distinguishing characteristic of the Montessori teaching method is A. children are grouped by developmental level rather than by age. B. children are free to select their own learning activities. C. it emphasizes emotional development and academic achievement. D. its focus on group activities and learning experiences.

B (In the Montessori classroom, children are encouraged to select their own activities; the teacher observes the children and assists them when they truly need help. There is very little didactic group teaching. The method is based on the assumption that the child is an active learner whose drive for self-development is aided by an orderly but stimulating environment. Regarding the other choices, "A is incorrect because the Montessori classroom is heterogeneous in regard to both age and developmental level -- older children are viewed as good role models for younger children. Choice "C is not an incorrect statement about the Montessori method, but it is not something that distinguishes it from other educational methods. And contrary to choice "D, critics of the Montessori method have cited its lack of opportunity for group learning experiences as one of its weaknesses.)

140. After losing several hundred dollars gambling at the casino, Jack decides to increase the amount he places on each subsequent bet in hopes of making his money back. This decision is best explained by: A. satisficing. B. loss aversion. C. bounded rationality. D. cognitive dissonance.

B (Kahneman and Tversky's -1979- Prospect Theory contains the notion of loss aversion which refers to the tendency to be influenced more by potential losses than potential gains or to base decisions more heavily on the fear of loss than the hope of gain. In contrast to commonly held beliefs that people are adverse to risk and make decisions based on logic, Kahneman and Tversky found that people are adverse to loss, not risk. This explains why gamblers will take riskier and riskier decisions after suffering a loss in hopes of making their money back, that is, to avoid realizing an actual loss. Satisficing -a.- refers to the decision-making style of using the minimal amount of information to reach a "good enough" solution. Bounded rationality -c.- is part of Herbert Simon's -administrative- model of decision making, which proposes that decision makers are not always completely rational in making choices. Instead, time and resources limit their consideration of alternatives, so they tend to consider alternatives only until a satisfactory one is identified. Cognitive dissonance -d.- theory predicts that, when we have two conflicting cognitions -e.g., I like the club but they don't want me"-, we'll be motivated to reduce the tension that this causes by changing one of our cognitions. )

7. Memory for the rules of logic and inference is part of A. procedural memory. B. semantic memory. C. episodic memory. D. read-only memory

B (Long-term memory has been divided into three components: semantic memory, procedural memory, and episodic memory. Semantic memory includes memory for the rules of logic and inference, as well as knowledge about language -e.g., what words mean and how they are used-. Procedural memory includes information about how to do things, such as how to drive a car. Episodic memory contains information about events that have been personally experienced. Read-only memory -ROM- is not part of long-term memory -- it is a computer term.)

150. A man is able to achieve erections during sleep, but, has difficulty achieving or maintaining erections during sexual activities. The most appropriate diagnosis would be: A. Hypoactive Sexual Desire Disorder B. Male Erectile Disorder C. Sexual Aversion Disorder D. Sexual Dysfunction Not Otherwise Specified

B (Male Erectile Disorder is characterized by a persistent or recurrent inability to attain or maintain an erection until completion of sexual activity. This best fits the description in this question. Hypoactive Sexual Desire Disorder -A- is diagnosed when a person has persistently deficient or absent sexual fantasies and desire for sexual activity. Since the question did not indicate that the man in this question lacked sexual desire, we should not assume that to be the case. Although, if a lack of desire was also present, both diagnoses would be given. There also is no evidence of extreme aversion to, or avoidance of, all genital contact with a sexual partner - which eliminates Sexual Aversion Disorder -C-. And Sexual Dysfunction NOS -D- is reserved for sexual dysfunctions that do not meet the criteria for any specific sexual dysfunction.)

210. According to Maslow, there are five levels of human needs that are represented in a hierarchical order. The stage that follows physiological needs is: A. order B. safety C. acceptance D. self-actualization

B (Maslow's five basic needs, arranged in hierarchical order of importance, are physiological, safety, social, esteem and self-actualization.)

205. A supervisee begins to unconsciously act toward her supervisor in similar ways as her own client acts towards her. This is an example of: A. Projective identification B. Parallel process C. Transference D. Identification.

B (Parallel process is a phenomenon in clinical supervision where the therapist in training behaves toward the supervising therapist in ways that mirror how the client is behaving toward the therapist in training.)

204. A person, in anticipation of failure, develops explanations and behavioral reactions to minimize ability deficits as possible attributions for the failure. This process is referred to as: A. Self-fulfilling prophecy B. Self-handicapping C. Self-perception theory D. Self-serving bias

B (People engage in self-handicapping strategies in order to avoid an internal attribution for failure. When a person anticipates failure, explanations and behavioral reactions are developed that minimize ability deficits as possible attributions for the failure. Self-fulfilling prophecy -a.- refers to a prediction made about some future behavior or event that modifies interactions so as to produce what is expected. Self-perception theory -c.- is the idea that people observe themselves in order to figure out the reasons they act as they do; people infer what their internal states are by perceiving how they are acting in a given situation. Self-serving bias -d.- refers to a class of attributional biases in which people tend to take credit for their successes and deny responsibility for their failures. )

221. According to Piaget, which of the following underlies cognitive development? A. biological maturation B. biological maturation and environmental stimulation C. social interactions D. changes in the ability to process, store, and retrieve information

B (Piaget proposed that cognitive development is the result of a combination of biological maturation and exposure to appropriate environmental stimuli. An implication of this assumption is that, even when a child is biologically ready for cognitive growth, it will not occur unless the child is also exposed to necessary environmental stimulation.)

160. Brain imaging techniques have identified abnormalities in which areas among people with Tourette Syndrome, Autistic Disorder, and Obsessive-Compulsive Disorder? A. frontal lobes and thalamus B. frontal lobes and basal ganglia C. parietal lobes and amygdala D. parietal lobes and thalamus

B (Research has consistently implicated pathology in the basal ganglia and frontal lobes in all three disorders.)

113. Employee creativity is impacted by personal and contextual characteristics. Creativity is diminished by: A. reduced density of work environment B. expecting a critical evaluation of work C. scoring high on a measure of openness to experience D. supportive supervision

B (Research investigating the impact of personal and contextual characteristics on the creativity of employees indicates increasing the physical distance between workers -a.-, providing supportive supervision -d.-, and being informed evaluations will be developmental rather than judgmental has a positive effect on employee creativity. The expectation that one's work will be critically evaluated tends to diminish creativity. Of the five personality factors, openness to experience -c.- has been most strongly related to creativity. See: Shalley, C., Zhou, J., & Oldham, G. -2004-. The effects of personal and contextual effects on creativity: Where should we go from here?, Journal of Management, 30-6-, 933-958.)

154. The best initial strategy for teaching complex motor skills that require speed and accuracy to be successfully performed is to: A. emphasize accuracy over speed B. emphasize speed over accuracy C. emphasize accuracy and speed equally D. emphasize an alternation between speed and accuracy

B (Research on speed and accuracy in learning complex motor skills suggests the best approach is to emphasize speed of performance initially although, to a certain degree, the optimal approach depends on the specific skill. -See: Engelhorn, R. -1997-. Speed and accuracy in the learning of a complex motor skills, Perceptual and Motor Skills, 85, 1011-1017.)

17. The best example of a secondary prevention program is A. a rehabilitation program. B. crisis intervention. C. a community education program. D. Head Start.

B (Secondary prevention involves early detection and intervention for a problem in order to reduce its duration and keep it from getting worse. Crisis intervention, suicide hotlines, and screening tests are all examples of secondary prevention.)

128. There is some evidence that hearing impairment in infancy -e.g., due to ear infections- may lead to some degree of persisting impairment in verbal skills and academic achievement even when the hearing deficit has been alleviated. This finding supports the notion of: A. canalization. B. sensitive periods. C. developmental delays. D. preordination.

B (Sensitive periods are times in development when a particular type of experience is necessary for development to occur. In the situation described in this question, a temporary loss of hearing during a sensitive period has had long-term repercussions. See M. Rutter, Continuities and discontinuities from infancy, in J. D. Osofsky -ed.-, Handbook of Infant Development, New York, John Wiley & Sons, 1987.)

172. Which of the following appear on Axis V of the DSM IV-TR? A. Personality Disorders B. Global Assessment of Functioning C. Psychosocial and Environmental Problems D. General Medical Conditions

B (The Global Assessment of Functioning -GAF- score ranges from 1-100 -zero indicates inadequate information to use the scale- and indicates the client's overall level of functioning, on a hypothetical continuum of mental health and illness. A score of 100 is superior functioning, and 1 indicates dangerously poor functioning. Personality Disorders -a.- appear on Axis II, Psychosocial and Environmental Problems -c.- appear on Axis IV and General medical conditions -d.- appear on Axis III. )

143. Psychodiagnostic tests such as the WAIS-III are commonly utilized to assist in the diagnosis of traumatic brain injury -TBI-. Which of the following WAIS-III factor indices is most likely to be impacted by traumatic brain injury? A. Perceptual Organization B. Processing Speed C. Verbal Comprehension D. Working Memory

B (The Processing Speed score is most negatively impacted by traumatic brain injury, especially for individuals with moderate to severe TBI. The Working Memory -d.- score tends to be less adversely affected than the Processing Speed score and Perceptual Organization is less affected than Processing Speed or Working Memory. -See, e.g., D. Fisher et al., WAIS-III and WMS-III profiles of mildly to severely brain-injured patients, Applied Neuropsychology, 2000, 7-3-, 126-132; and P. van der Heijden and J. Donders, WAIS-III factor index score patterns after traumatic brain injury, Assessment, 2003, 10-2-, 115-122.)

180. Which of the following is not a proposed axis for further study according to the DSM-IV-TR? A. Social and Occupational Functioning Assessment Scale B. Sickness Impact Profile C. Defensive Functioning Scale D. Global Assessment of Relational Functioning Scale

B (The Sickness Impact Profile -SIP- is a quality of life measure used to assess the impact of disease on both physical and emotional functioning. The self, or interviewer, administered items address activities of daily living with the score indicating the level of dysfunction. The higher the score, the greater the level of dysfunction. The Social and Occupational Functioning Assessment Scale, or SOFAS, focuses exclusively on an individual's level of social and occupational functioning, typically the functioning for the current time period. The Defensive Functioning Scale, or DFS, reflects the defenses or coping styles exhibited by an individual at the time of evaluation. The Global Assessment of Relational Functioning Scale, or GARFS, uses a hypothetical continuum, ranging from optimal to dysfunctional, to rate the overall functioning of a family or relationship. It is similar to the DSM-IV Axis V - Global Assessment of Functioning Scale.)

44. Which of the following medications is least likely to cause anticholinergic symptoms? A. Imipramine B. Fluoxetine C. Amitriptyline D. Venlafaxine

B (The anticholinergic effects, which include dry mouth, blurred vision, constipation, urinary retention and tachycardia, are much more likely to result from the tricylics or heterocyclics than the SSRI's. Of the choices listed only Fluoxetine -Prozac- is an SSRI. Imipramine -Tofranil- and Amitriptyline -Elavil- are tricyclics and Venlafaxine -Effexor- is a heterocyclic.)

223. The cingulate cortex is most associated with: A. spatial memory B. emotions and motivation C. somatosensory processing D. balance and posture

B (The cingulate cortex is part of the limbic system and is believed to play an excitatory role in emotions and in motivating behaviors. It's also known as the satisfaction center - mediating feelings of satisfaction following eating and sex.)

174. The ability to unconsciously monitor the contents of one conversation while consciously focusing on another is referred to as: A. change blindness B. the cocktail-party phenomenon C. exogenous attention D. endogenous attention

B (The cocktail-party phenomenon is a characteristic of selective attention which indicates that even when you are intently focusing on one conversation and unaware of another, the mention of your name will immediately get your attention. Change blindness -a.- refers to difficulty perceiving major changes to unattended-to parts of a visual image when the changes are introduced during brief interruptions in the presentation of the image -See: Rensink, R.A. -2005-. Change Blindness. In L. Itti, G. Rees, and J.K. Tsotsos -eds-. Neurobiology of Attention. -pp. 76-81-. San Diego, CA: Elsevier.- Exogenous attention -c.- refers to the automatic attraction of attention, due to, among others, the sudden appearance of a stimulus. This is a typical bottom-up process, controlled by external stimulus presentation, and not under subjects' control. Endogenous attention -d.- refers to a typical top-down, attentional effort under control of the individual, for example, when attention is being focused on the basis of instructions.)

49. For children of divorce, frequent visitation with the noncustodial father usually results in: A. fewer behavioral problems among boys but not girls and higher academic achievement among girls but not boys B. fewer behavioral problems and higher academic achievement if the noncustodial father is supportive and authoritative C. fewer behavioral problems and higher academic achievement regardless of the characteristics of the noncustodial father D. fewer behavioral problems and higher academic achievement regardless of the parent's level of conflict

B (The effects of visitation on children's adjustment following divorce are dependent upon several factors, including the quality of the relationship between the divorced parents and certain attributes and behaviors of the noncustodial parent. Specifically, frequent visitation with the noncustodial father has been found to result in fewer behavioral problems and higher academic achievement, especially in boys, when the father was supportive, authoritative, and lacked any significant problems in adjustment, and when the child was not exposed to high levels of conflict between the parents See: E. M. Hetherington, An overview of the Virginia Longitudinal Study of Divorce and Remarriage with a focus on early adolescence, Journal of Family Psychology,7-1-, 1993, 39-56)

131. Which of the following forebrain structures are located around the brain stem and are important for motivated and emotional behavior? A. basal ganglia B. limbic system C. tegmentum D. reticular formation

B (The limbic system is a set of structures controlling motivated and emotional behaviors such as eating, drinking, sexual behavior and aggression. Main structures of the limbic system include the olfactory bulb, the hypothalamus which regulates motivational behaviors, the pituitary gland which controls the timing and amount of hormone release, and the hippocampus which is associated with learning and memory. The basal ganglia -a.- codes and relays information associated with the control of voluntary movement. The tegmentum -c.- is one of two regions of the "midbrain" and contains the substantia nigra which is part of the brain's sensorimotor system. The reticular formation -d.- is a group of nerve fibers located inside the brainstem which controls sleep, arousal, and attention. )

85. Approximately what percent of women experience full-blown postpartum -clinical- depression? A. 1 to 5%. B. 10 to 20%. C. 20 to 30%. D. 30 to 40%.

B (The majority of women experience some depression following childbirth but, for most, these symptoms are mild. For about 10 to 20% of women, symptoms are sufficiently severe to qualify for a diagnosis of Major Depression.)

196. When constructing an achievement test, which of the following would be useful for comparing total test scores of a sample of examinees to the proportion of examinees who answer each item correctly? A. classical test theory B. item response theory C. generalizability theory D. item utility theory

B (The question describes the kind of information that is provided in an item response curve, which is constructed for each item to determine its characteristics when using item response theory as the basis for test development. -Note that there is no such thing as "item utility theory.")

219. Which of the following correlation coefficients is used to assess convergent validity: A. heterotrait-monomethod B. monotrait-heteromethod C. heterotrait-heteromethod D. monotrait-monomethod

B (The response choices make up a multitrait-multimethod matrix, a complicated method for assessing convergent and discriminant validity. Convergent validity requires that different ways of measuring the same trait yield the same result. Monotrait-heteromethod coefficients are correlations between two measures that assess the same trait using different methods; therefore if a test has convergent validity, this correlation should be high. Heterotrait-monomethod and heterotrait-heteromethod both confirm discriminatory validity, and monotrait-monomethod coefficients are reliability coefficients.)

79. According to the results of meta-analyses, which of the following statements regarding gender and leadership style is true? A. Men tend to use more of a participatory leadership style, while women tend to use more of an autocratic style. B. Women tend to use more of a participatory leadership style, while men tend to use more of an autocratic style. C. Men tend to use more of an autocratic style of leadership, but no general statements about women's typical leadership style can be made. D. There are no overall differences between the sexes in terms of leadership style.

B (The results of meta-analyses suggest that there are slight but statistically significant overall differences between men and women in leadership style. Specifically, women tend to adopt a more democratic or participatory leadership style, characterized by a greater concern with maintenance of interpersonal relationships and task accomplishments. Men, on the other hand, tend to adopt a more autocratic style.)

58. An individual taking clozapine begins exhibiting symptoms of muscle rigidity, tachycardia, hyperthermia, altered consciousness, and autonomic dysfunction. In this case, the best course of action would be to: A. reduce the dose gradually until the symptoms are alleviated B. stop the drug immediately and administer electrolytes and fluids C. switch to a traditional antipsychotic drug D. check to see what other drugs the patient is taking since clozapine does not produce these symptoms

B (The symptoms listed are characteristic of neuroleptic malignant syndrome -NMS-, which is a rare, but serious and potentially lethal syndrome that can result from the use of any neuroleptic or antipsychotic drugs. The syndrome commonly develops within the first 2 weeks of treatment in most cases however it may develop any time during drug therapy and can also occur in people taking anti-Parkinsonism drugs known as dopaminergics if those drugs are discontinued abruptly. All antipsychotics, typical or atypical, may precipitate the syndrome although potent neuroleptics -e.g., haloperidol, fluphenazine- are more frequently associated with NMS. Other agents associated with NMS include prochlorperazine -Compazine-, promethazine -Phenergan-, clozapine -Clozaril-, and risperidone -Risperdal- as well as non-neuroleptic agents that block central dopamine pathways such as metoclopramide -Reglan-, amoxapine -Ascendin-, and lithium. Generally, intensive care is needed. The neuroleptic or antipsychotic drug is discontinued, and the fever is treated aggressively. A muscle relaxant may be prescribed. Dopaminergic drugs, such as a dopamine agonist, have been reported to be useful.)

162. Narcissistic, Borderline, and Histrionic Personality Disorders share in common which of the following characteristics? A. irresponsibility and impulsivity B. affective instability C. recurrent suicidal threats D. a grandiose sense of self

B (The three Personality Disorders listed are all characterized by dramatic, emotional, and/or erratic behaviors. Affective instability is the characteristic shared by all three disorders. Impulsivity and recurrent suicide threats are characteristics of Borderline Personality Disorder only; and a grandiose sense of self describes Narcissistic Personality Disorder, but not the other two disorders -although both do involve disturbances in the sense of self-.)

173. In terms of interviews as selection techniques, which of the following is most consistent with the research? A. Panel interviews generally have the highest levels of validity, and they are especially valid when an average -versus consensus- rating is derived. B. When interviewers are given biodata information about an interviewee prior to the interview, interviewers give less credence to interview information when the biodata is not supportive of a decision to hire than when the biodata is very supportive of a decision to hire. C. Although the research is inconsistent, the best conclusion about interviews is that future-oriented -situational- interviews are more valid than past-oriented -behavior description- interviews regardless of whether the interview is structured or unstructured. D. One of the difficulties with interviews, even when they are structured, is that they are highly susceptible to gender biases, especially when the interviewee and the interviewer are of different genders.

B (There is some evidence that interviewers place less importance on interview information when biodata is not very favorable and more importance when it is supportive of a hiring decision. Apparently, a good interview cannot make up for a bad history, but a good history can be supported or canceled out by the results of the interview.)

155. According to recent meta-analyses of child psychotherapy outcome studies, which of the following statements is most true? A. There are no differences between the effectiveness of behavioral and non-behavioral interventions in the treatment of children. B. Girls respond better to psychotherapy than boys, with adolescent girls responding best. C. Girls respond better to psychotherapy than boys, with younger girls responding best. D. At all age levels, boys respond better to psychotherapy than girls.

B (This is one of the many areas where the results of research are contradictory and somewhat controversial. However, the most recent meta-analyses of research in this area have found that across treatment approaches, girls respond better than boys, with adolescent girls responding best of all. This is somewhat contradictory to earlier research, which found that younger children respond better than older children. Because the findings of research sometimes contradict each other, it can be frustrating to decide which is the best answer to questions such as these. Of course, you'd want to find an answer that is consistent -or not inconsistent, at least- with the results of all the research. However, if such a response is not available, you should go with the results of more recent research.)

104. Herbert Simon's decision-making model suggests that decision makers A. consider all alternatives and then pick the best one. B. consider alternatives only until they find one that seems acceptable. C. consider only those alternatives that have a 50% or better chance of success. D. rely more on their affective than cognitive reactions when choosing an alternative.

B (To answer this question, you have to have the name Herbert Simon linked with the bounded rationality -administrative- model of decision making, which proposes that decision makers are not always completely rational in making choices. Instead, time and resources limit their consideration of alternatives, so they tend to consider alternatives only until a satisfactory one is identified.)

96. The basic requirements of a token economy are: A. stimulus sensitization, choice of tokens, rate of exchange. B. target behaviors, choice of reinforcers, rate of exchange. C. goal setting, staff cooperation, choice of reinforcers. D. target behaviors, choice of tokens, primary reinforcers.

B (To institute a token economy program, you need to know the behaviors you want to change -the target behaviors-. You also need to know what is reinforcing for the client -choice of reinforcer-. For a hospitalized schizophrenic, it might be walking around; for a child, it might be a candy treat. You also need to know the relationship between token and the reinforcer -rate of exchange-; that is, how many tokens will purchase the reinforcer.)

176. If you are interested in determining whether the relationship between arousal and performance assumes a linear or a non-linear shape, the best statistical analysis to use would be A. multiple regression analysis. B. trend analysis. C. logistic regression. D. principal components analysis.

B (Trend analysis is a statistical technique used to determine the trend or shape that best describes the relationship between two variables. The technique basically involves collecting data on two variables and running statistical analyses to determine what trend or trends -e.g., linear, U-shaped- are significant. For example, in studying the relationship between arousal and performance, one could study 100 students and collect data on how aroused they are and how well they perform. Then, one could run a separate analysis for different types of trends and see which receives the strongest support.)

116. All of the following are measures of variability except: A. variance B. standard error C. range D. standard deviation

B (Variability represents the amount of difference found in responses from a population or sample on a topic being investigated. Variance -a.-, range -c.-, and standard deviation -d.- all reflect the variability in the data. Standard error -of measurement- is not a measure of variability. It is a statistic indicating the amount of difference in results that is accounted for by flaws or "noise" in the instrument used to measure a variable.)

215. Differing locus of control and locus of responsibility combinations yield the four worldviews described by Sue and Sue -2003-. Mainstream American culture would best be characterized by an: A. internal locus of control and external locus of responsibility B. internal locus of control and internal locus of responsibility C. external locus of control and internal locus of responsibility D. external locus of control and external locus of responsibility

B (Worldview refers to the manner in which people perceive, evaluate and react to encountered situations. Sue and Sue describe mainstream American culture "as the epitome of the individual-centered approach that emphasizes uniqueness, independence and self-reliance" -p.277-. This is characterized by an internal locus of control and locus of responsibility in Sue, D.W. & Sue, D. -2003-. Counseling the culturally diverse: Theory and practice. 4th edition. New York: John Wiley.)

121. You are a small town psychologist in private practice and a client has written a letter to the Psychology Board complaining that you have a very unprofessional staff. The Board has now written you to inform you of this matter. You should: A. Discipline your staff B. Respond to the Board C. Call the client and let them know that you will make the appropriate changes with your staff D. Refer the client to a psychologist in another town

B (You must respond to the Board. To not do so is itself an ethical violation. While it is possible that this is a clinical issue between yourself and your client it is no longer in the therapeutic realm. Your client has gone to an outside source and that source is concerned enough to begin a preliminary investigation.)

34. A child's ability to maintain a mental picture of a toy even after it is removed is called: A. flashbulb memory B. method of loci C. eidetic imagery D. sustained attention

C ( Eidetic imagery, or photographic memory, is associated with improved ability to memorize information and tends to be more common in children. Another strategy for improving recall using visual images, is the method of loci -b.-, which involves associating items to be remembered with mental images of places or specific items. This is useful for recalling information in a specific order. Flashbulb memory -a.- describes vivid, detailed memories of emotionally-charged or surprising events. Sustained attention -d.- is the ability to direct and focus cognitive activity on specific stimuli over an extended period.)

144. A Performance IQ score that is 15 points higher than the Verbal IQ score on the WAIS-III is suggestive of all of the following except A. learning disabilities. B. low socioeconomic status. C. depression. D. antisocial behavior.

C (A Verbal-Performance IQ discrepancy on the Wechsler test is considered significant when it is 12 points or more. There are a number of reasons why the Performance IQ may be higher than the Verbal IQ. The factors listed in answers A, B, and D may account for a higher Performance IQ, while depression -answer C- is a possibility when the Verbal IQ is higher.)

123. A predictor with a criterion-related validity coefficient of .40 will be most useful when there is a A. high selection ratio and moderate base rate. B. high selection ratio and low base rate. C. low selection ratio and moderate base rate. D. low selection ratio and low base rate.

C (A low selection ratio means that there's lots of applicant to choose from -which is preferable to having only a few to choose from-. A moderate base rate is preferable to a high or low base rate because this means that there's room for improvement in the selection process. When the base rate is high, the company is already doing a good job and doesn't need a new predictor; when the base rate is low, this usually means that something other than selection is the problem -e.g., the company's performance standards are too high or there just aren't enough good applicants to choose from-.)

200. A number of books in the popular press have been written regarding the relationship between psychological factors and cancer. Which of the following statements best reflects the outcome of scientific studies of this issue? A. Psychological factors are related both to the onset of cancer and the success of recovery from it. B. Psychological factors are related to the onset of cancer but not to the success of recovery from it. C. Psychological factors are not related to the onset of cancer but are related to the success of recovery from it. D. Psychological factors are related to neither the onset of cancer nor the success of recovery from it.

C (A number of theories regarding the relationship between psychological factors and the onset of cancer have been proposed. For instance, some authors have proposed that the "Type C personality, typically described as a cooperative, unassertive patient who suppresses anger and complies with external authorities, is at a higher risk for cancer. However, most research shows that psychological factors and stressful events have a small or no effect on cancer incidence. By contrast, psychological factors do appear to be related to recovery from cancer. For instance, psychological treatments combining support and training in self-hypnosis are associated with higher survival rates and improved quality of life in cancer patients.)

33. Which of the following statements is most consistent with Lewin's field theory? A. As a person moves towards one of the goals in an approach-approach conflict, it becomes less attractive and the other goal becomes more attractive. B. Leadership is a function of the relationship between a task and the environment. C. Behavior is a function of the relationship between the person and the environment. D. A person's "life space" is equivalent to Jung's notion of the collective unconscious.

C (According the Lewin's field theory, behavior is a function of the relationship between a person and his or her environment. Lewin used the following formula to express this relationship: B = f-P,E- where B is behavior, P is the person, and E is the environment. Choice "A is the opposite of Lewin's prediction. That is, when faced with an approach-approach conflict, the selected choice becomes more attractive while the other choice becomes less attractive. "Life space" is also a central concept in field theory but refers to everything in a person's psychological environment - not the collective unconscious.)

167. Which of the following statements regarding children with Oppositional Defiant Disorder is most true? A. Symptoms of the disorder are usually present in all settings and with all people. B. Symptoms of the disorder are more likely to be present when the child is with people he or she doesn't know very well than when the child is around familiar people. C. Symptoms of the disorder are more likely to be present when the child is around familiar people than when the child is with people he or she doesn't know that well. D. The disorder is characterized by unexpected and unpredictable symptoms, so, although symptoms of the disorder occur infrequently, they are just as likely to occur with familiar people as with strangers.

C (According to DSM-IV, manifestations of Oppositional Defiant Disorder are almost invariably present in the home setting and in interactions with adults or peers whom the individual knows well. They may not be apparent in school or community settings or in interactions with people the individual does not know that well. For this reason, symptoms of the disorder are often not present during the initial clinical examination.)

127. All of the following would be standard aspects in the assessment and diagnosis of Dementia of the Alzheimer's Type, except A. administering a mental status exam. B. administering tests of neuropsychological functioning. C. administering medical tests designed to identify the presence of the Alzheimer's gene. D. using neuroimaging techniques such as magnetic resonance imaging -MRI-.

C (Although some progress has been made recently in identifying genetic markers of Alzheimer's Disease, there is no diagnostic test currently available that can isolate an Alzheimer's gene. Indeed, direct evidence that a person has this disease is hard to come by. Instead, the diagnostic process involves determining that the diagnostic criteria for the disorder are met, the typical features of Alzheimer's are present, and other possible causes of the disturbance have been ruled out. Choices A, B, and D all describe techniques that are typically part of this diagnostic process.)

147. Which of the following would most likely be remembered? A. a political advertisement expressing views consistent with the viewer's beliefs B. a political advertisement expressing views opposite to the viewer's beliefs C. an emotionally-charged event D. an emotionally-charged event which was expressively suppressed

C (As you may have guessed, memory is enhanced for emotionally-charged events. Research has also found that this enhancement is not solely due to increased attention -S. Christianson, E. Loftus, H. Hoffman, & G. Loftus. Eye Fixations and Memory for Emotional Events, Journal of Experimental Psychology: Learning, Memory, and Cognition, 1991, 17-4-, 693-701-. Attempts to suppress emotion -D- have been found to decrease one's memory for the event. Examples of emotional suppression include biting one's lip to keep from crying or maintaining a poker face after being dealt a great hand of cards -J. Richards & J. Gross. Emotion regulation and memory, Journal of Personality and Social Psychology, 2000, 79-3-, 410-424-.)

119. ___________________ may result in a job performance measure having low validity, even though it is reliable. A. differential validity B. criterion contamination C. criterion deficiency D. researcher deficiency

C (Criterion deficiency refers to what is missed or deficient in the criterion used. For example, if typing speed is used as the sole criterion for determining successful job performance by a secretary, it would be a deficient criterion, since typing speed is only one of several skills needed to be a successful secretary. Differential validity -"A- refers to a test which has significantly different validity coefficients for different subgroups. Criterion contamination -"B- occurs when a rater's knowledge of an employee's performance on a predictor biases how the employee is rated on a criterion.)

197. The Treatment of Depression Collaborative Research Program -TDCRP-, conducted by the National Institute of Mental Health, compared Cognitive Therapy, antidepressant medication, pill placebo, and interpersonal psychotherapy treatments in depressed outpatients. The results of this study found: A. cognitive therapy is more effective than the other treatments for mild to moderate depression B. cognitive therapy is more effective than the other treatments for moderate to severe depression C. no difference in observed outcome between cognitive therapy and antidepressant medications -ADMs- for mild to moderate depression. D. no difference in observed outcome between cognitive therapy and antidepressant medications -ADMs- for moderate to severe depression.

C (Findings of the NIMH's TDCRP research project indicate no differences in outcome were observed between CT and ADM among all patients. In other words, overall, cognitive therapy -CT- and antidepressant medication are about equally effective in the treatment of depression. In a secondary analysis of more severely depressed patients, however, ADM outcomes were superior to both cognitive therapy and placebo. Antidepressant medications -ADMs- are the most widely used treatment for major depressive disorder -MDD- in the United States and evidence supports the efficacy of ADMs, particularly among more severely depressed patients. as first-line therapy for patients with moderate to severe MDD. -See: DeRubeis, R.J., Hollon, S.D., Amsterdam, J.D., Shelton, R.C., Young, P.R., Salomon, R.M., O'Reardon, J.P., Lovett, M.L., Gladis, M.M., Brown, L.L., & Gallop,R. -2005- Cognitive Therapy vs Medications in the Treatment of Moderate to Severe Depression. Archives of General Psychiatry.62-4-:409-416.)

152. You see an elderly male in therapy. In the first session, he breaks down sobbing and admits that he has recently "helpeD his wife, who was suffering from incurable cancer, to die. First you need to A. comfort him as best you can and make a call to the police. B. get the number of a trusted family member and alert them. C. assess your client's potential for suicide. D. suggest that your client call the police while still in your office.

C (First of all, this question is not about a Tarasoff situation. You are speaking to the husband after the fact of the wife's death so you do not have a mandated breach of confidentiality, and instead now need to decide what is in the best interest of your client. What you have is a bereaved client who is at a high risk for suicide. With his best interests and welfare in mind, you need to assess his potential for suicide and then decide if you want to get the number of a trusted family member and alert them -choice B-. You would not be able to breach confidentiality as suggested in answer A and D for the purpose of "turning in" your client.)

16. From a legal standpoint, a charge of malpractice against a psychologist would be held valid if it were proven that the therapist: A. had a duty of care to the patient, held malevolent intentions toward the patient, and engaged in activity that resulted in harm to the patient. B. had a duty of care to the patient and held malevolent intentions toward the patient; demonstration of actual harm is not necessary. C. had a duty of care to the patient and engaged in activity that resulted in harm to the patient; malevolent intentions need not be present. D. engages in activity that harms a patient; malevolent intentions and a duty of care are not necessary.

C (For a malpractice claim against a psychologist to be held valid, three elements must be proven: 1- the psychologist must have had a professional relationship with -and, therefore a duty of care to- the client; 2- the psychologist must have been negligent or failed to live up to that duty; and 3- harm to the patient must have resulted. In other words, the psychologist's intentions are not an issue.)

112. Delirium may occur due to intoxication with all of the following substances except A. cocaine. B. cannabis. C. caffeine. D. LSD.

C (Hallucinogens -e.g., LSD-, cocaine, and cannabis -- as well as alcohol, amphetamines, inhalants, opioids, PCP, and sedatives -- may produce Intoxication Delirium. Caffeine, however, does not cause delirium.)

133. Huntington's Disease is most associated with decreased amounts of: A. dopamine B. epinephrine C. GABA D. norepinephrine

C (Huntington's Disease is believed to begin when cells within the striatum -caudate and putamen- of the basal ganglia begin to be destroyed. The striatum is responsible for producing GABA, which regulates the levels of dopamine in the brain through an inhibitory process. The death of the striatum cells causes decreased amounts of GABA which leads to an overproduction of dopamine and results in chorea -uncontrollable and irregular muscle movements, especially of the arms, legs, and face-.)

129. At the beginning of group therapy, all members signed a contract to maintain confidentiality. If one of the members breaks confidentiality it is: A. an ethical violation B. a legal violation C. a moral violation D. an ethical, moral and possible legal violation

C (In the context of group therapy, a moral responsibility to respect confidentiality extends beyond the therapist to each group participant. However, while there may be a moral obligation to maintain confidentiality, there isn't an ethically enforceable obligation. The Ethics Code, Standard 4.02, addresses the limitations of confidentiality and Standard 10.01, Informed Consent in Therapy, addresses the need to discuss the limits of confidentiality at the beginning of -group- therapy. In group work agreements, therapists clearly define confidentiality and the parameters for the specific group being entered, explain the importance, and discuss the difficulties related to confidentiality involved in group therapy. Since confidentiality cannot be guaranteed by the therapist, group members have provided "implied consent" to the release of the information by voluntarily agreeing to participate. In most states, group members are not legally obligated to maintain confidences.)

109. A young man is in a car accident and apparently receives some brain injury. Two days later, the man has a 10 a.m. appointment with a neurologist to evaluate the extent of his injury. Later that same day, the young man encounters the neurologist in a coffee shop but doesn't recall the neurologist, their meeting, or the evaluation. The young man is apparently suffering from A. paraprosopia. B. prosopagnosia. C. anterograde amnesia. D. retrograde amnesia.

C (In this situation, the patient seems to be having trouble forming new memories. This is referred to as anterograde amnesia. Paraprosopia -answer A- involves visual hallucinations of terrifying faces; prosopagnosia -answer B- is the inability to recognize familiar faces.)

148. Research on the use of mental health services by members of minority groups indicates that: A. utilization rates are lower for members of all minority groups than for whites. B. utilization rates are higher for members of all minority groups than for whites. C. utilization rates are higher for members of some minority groups and lower for members of other minority groups than for whites. D. utilization rates for members of minority groups are about the same as the rates for whites.

C (It's difficult to draw any general conclusions about utilization rates because the research findings are inconsistent. However, a 1991 study by Sue et al. -which is frequently cited in the literature- reports underutilization by Asian-and Latino-Americans and overutilization by African-Americans. The results of other studies also suggest that there are group differences in utilization rates, so response C is the best.)

151. Increasing internal validity is best achieved by: A. random selection B. matching C. random assignment D. blocking

C (Known as the "great equalizer," randomization of subjects to groups is the most powerful way for controlling extraneous variables. Unlike random assignment which occurs after subjects are selected, random selection refers to a method of selecting subjects to participate from the population being studied. Random selection influences external validity. Matching, a procedure to ensure equivalency on a specific extraneous variable, and blocking, studying the effects of the extraneous variable, are also methods of increasing internal v)

170. The experience of REM-rebound occurs most often when A. barbiturates are used chronically B. a person begins using sedatives. C. the use of sedatives is discontinued. D. alcohol is substituted for barbiturates.

C (Most drugs suppress REM sleep. When REM sleep is suppressed, a sudden rebound effect occurs soon after the removal of the suppressing agent.©

189. Studies on gender differences in physical development suggest that, until about age _____, girls should be able to compete effectively with boys in baseball and other sports. A. 7 B. 10 C. 12 D. 15

C (Most physical differences between boys and girls do not become prominent until puberty. Before puberty, boys and girls are about equal, for instance, in terms of speed and strength, implying that they should do about equally well in many sports.)

185. Which of the following techniques would be most useful for combining test scores when poor performance on one test can be offset by excellent performance on another: A. multiple baseline B. multiple hurdle C. multiple regression D. multiple cutoff

C (Multiple regression is the preferred technique for combining test scores in this situation as it is a compensatory technique since a low score on one test can be offset -compensated for- by high scores on other tests. Multiple baseline -a.- is a research design, not a method for combining test scores. Multiple hurdle -b.- and multiple cutoff -d.- are noncompensatory techniques.)

108. Following a stroke or head trauma, which of the following memory functions is most likely to be affected? A. iconic memory B. long-term store C. memory for newly learned information D. verbal memory

C (Questions like this can be frustrating because it is really impossible to make blanket generalizations. When the brain is compromised by a medical problem -or by substance use-, the nature of impairment always depends on the location in the brain of damage and the extent of damage. However, if memory is impaired, the ability to retain newly learned information is most likely to be affected. Impairment in other types of memory, such as long-term episodic memory or verbal -semantic- memory, usually indicates more severe memory damage in which impairment in learning new information is also present.)

111. Depth perception in infants develops in which of the following sequences? A. kinetic, pictorial, binocular B. pictorial, binocular, kinetic C. kinetic, binocular, pictorial D. binocular, kinetic, pictorial

C (Research has indicated that infants develop depth perception in a predictable sequence: kinetic, binocular, and pictorial. Kinetic depth cues are based on movements of objects in the environment or the body. Babies as young as 3 weeks begin perceiving kinetic cues. For example, they will blink their eyes defensively when an object approaches their face and looks as if its going to hit them. Binocular depth perception begins between 2 and 3 months. And pictorial depth cues, which are used by artists to make paintings look three-dimensional, begins to develop at about 7 months of age.)

75. According to the current research, the effects on children of observing aggressive models, such as through television violence, are A. short-term but not long-term. B. long-term but not short-term. C. both short-term and long-term. D. neither short-term nor long-term.

C (Research on the effects of television violence on children's behavior demonstrates that viewing aggressive models is associated with increased aggressive behavior. Moreover, these effects can be long-lasting; they have been observed in longitudinal studies lasting up to 22 years.)

183. The age of onset of schizophrenia is: A. about the same time for men and women. B. usually earlier for women than for men. C. usually earlier for men than for women. D. never after the age of 30.

C (Schizophrenia has been found in about 1 in every 100 people around the world in many studies. Symptoms commonly begin in late adolescence or early adulthood. Schizophrenia with an onset in adolescence -prior to age 18- is less common and childhood onset -before age 13- is exceedingly rare. According to the DSM-IV-TR, the age at onset differs between the sexes with males having an earlier onset of between 18 and 25 years and females between 25 and the mid-30s. Note the age-at-onset distribution is bimodal for women, with a second peak occurring later in life -i.e., 3-10% have onset after age 40-, however unimodal for males. )

103. Form A is administered to a group of employees in Spring and then again in Fall. Using this method, what type of reliability is measured? A. split-half B. equivalence C. stability D. internal consistency

C (Test-retest reliability, or the coefficient of stability, involves administering the same test to the same group on two occasions and then correlating the scores. Alternative forms reliability, or coefficient of equivalence -response "B-, consists of administering two alternate forms of a test to the same group and then correlating the scores. Internal consistency reliability -response "D- utilizes a single test administration and involves obtaining correlations among individual test items. Split-half reliability -response "A- is a method of determining internal consistency reliability.)

222. According to the DSM-IV-TR, Bipolar I Disorder is differentiated from Bipolar II Disorder by: A. at least one Major Depressive Episode and one Mixed Episode without a Manic or Hypomanic Episode B. at least one Major Depressive Episode and one Hypomanic Episode without a Manic or Mixed Episode C. one or more Manic or Mixed Episodes D. one Mixed Episode or Hypomanic Episode

C (The DSM diagnosis for Bipolar II Disorder requires the history or presence of Major Depressive and Hypomanic Episodes -b.- in the absence of Manic or Mixed Episodes -a., d.-. If an individual diagnosed with Bipolar II Disorder develops a Manic or Mixed Episode then the diagnosis is changed to Bipolar I Disorder.)

156. What percent of individuals diagnosed of Panic Disorder also have Agoraphobia in community samples according to the DSM-IV-TR? A. less than 10% B. 15-25% C. 33-50% D. 50-66%

C (The DSM-IV-TR reports prevalence rates for Panic Disorder of 1 to 2% in community samples and states that "approximately one-third to one-half of individuals diagnosed with Panic Disorder in community samples also have AgoraphobiA -p. 436-.)

163. Orientation is most frequently measured by which of the following scales? A. Global Orientation and Amnesia Test B. Gross Orientation and Awareness Test C. Galveston Orientation and Amnesia Test D. Gollingberg Orientation and Awareness Test

C (The Galveston Orientation and Amnesia Test -GOAT-, which assesses temporal orientation primarily, was developed to serially evaluate cognition during the subacute stage of recovery from closed head injury. The scale measures orientation to person, place, and time, and memory for events preceding and following the injury.)

191. The WAIS-III measures working memory with which of the following subtests? A. matrix reasoning B. block design C. arithmetic D. digit symbol-coding

C (The arithmetic subtest along with the digit span subtest and letter-number sequencing subtest are used as measures of the Working Memory Factor. Matrix reasoning -answer A- and block design -answer B- are part of the Perceptual Organization Factor. Digit-symbol coding -answer D- is included in the Perceptual Speed Factor.)

141. A psychologist is asked by the attorney for the father in a divorce procedure to determine, in his expert opinion, which parent is better able to care for the children. The father is in the same state as the psychologist but the mother is in a distant state. The psychologist should A. agree to evaluate the father and give his opinion as requested as long as the conclusions are not influenced by who pays the fee. B. evaluate the father but not send the report until asked for it by the judge. C. evaluate the children and both parents before giving such an opinion. D. refuse to accept this work since there can be no firm predictions made as to which parent would be better able to care for the children.

C (The basic principle is that you can't give an opinion as to which parent will be better able to care for the children unless you interview both parents and the children. Otherwise, all you can attest to is your findings about the psychological health of whomever you evaluate. You can't, if you only see one of the parents, opine as to which one is better: You'd have no basis to come to such a conclusion.)

194. A self-fulfilling prophecy where subordinates perform better when expected to do so by their superiors is known as: A. the Barnum effect B. Unconscious inference C. Pygmalion effect D. Tend-and-befriend response

C (The question describes the Pygmalion effect. The Barnum effect -a.-, also known as the Forer effect, is defined as the tendency for people to accept very vague or general feedback, such as horoscopes, as accurate. Unconscious inference -b.- is Helmholtz's term for perception that occurs outside of conscious awareness. The tend-and-befriend response is hypothesized to be typical for females; stressors prompt females to protect their offspring and join social groups to reduce vulnerability. )

166. A lab subject heard a woman in the next lab fall and cry for help. In this "lady in distress" experiment, a person was least likely to help when tested with: A. no others present. B. a stranger. C. a passive confederate. D. a friend.

C (The subject is most likely to rush to aid someone in distress when all alone in the lab; less likely when there is a stranger or a friend in the room; and least likely when there is a passive confederate in the same room who has been instructed to ignore the whole thing. There is no "diffusion of responsibility" or "pluralistic ignorance" when the subject is alone.)

105. In some plagiarism cases, writers may be using what they perceive to be their original ideas, consequently failing to credit the ideas to the proper source, while they are actually retrieved thoughts from memory. This is an example of a phenomenon known as: A. source amnesia B. source misattribution C. cryptomnesia D. false fame effect

C (There are several reasons for errors in memory. Source misattributions -b.- occur when individuals misremember the time, place, person, or circumstances involved with a memory. Cryptomnesia -c.- is an example which occurs when a person perceives the recovery of information from memory as being an original idea of their own. Another example of source misattribution is the false fame effect -d.- in which subjects remembered the names but could not recall where they had encountered the names so they concluded that the individuals were famous. Source amnesia -a.- is an episodic memory disorder where source or contextual information surrounding facts are severely distorted and/or unable to be recalled. An individual remembers some factual information, yet forgets the contextual information related to the fact such as when, where, and with whom the fact was learned.)

187. Which of the following methods of establishing a test's reliability is, all other things being equal, likely to be lowest? A. split-half B. Cronbach's alpha C. alternate forms D. test-retest

C (You probably remember that the alternate forms coefficient is considered by many to be the best reliability coefficient to use when practical -if you don't, commit this factoid to memory now-. Everything else being equal, it is also likely to have a lower magnitude than the other types of reliability coefficients. The reason for this is similar to the reason why it is considered the best one to use. To obtain an alternate forms coefficient, one must administer two forms of the same test to a group of examinees, and correlate scores on the two forms. The two forms of the test are administered at different times and -because they are different forms- contain different items or content. In other words, there are two sources of error -or factors that could lower the coefficient- for the alternate forms coefficient: the time interval and different content -in technical terms, these sources of error are referred to respectively as "time sampling" and "content sampling"-. The alternate forms coefficient is considered the best reliability coefficient by many because, for it to be high, the test must demonstrate consistency across both a time interval and different content.)

178. A primary reinforcer is the same as: A. A conditioned reinforcer B. The first reinforcer used C. Pseudoconditioning D. An unconditioned reinforcer

D (A primary reinforcer is the same as an unconditioned reinforcer. These are items that acquire their reinforcing value without special training. Food and water are examples of primary reinforcers.)

175. According to the theory of psychopathology on which Beck's model of cognitive therapy is based, which of the following is true regarding the role of schema? A. Dysfunctional schemas always cause problems, even if they are not associated with symptomatic behavior. B. Certain types of schema, known as "core schemA are not accessible to the conscious mind. C. Schemas govern cognition but not other psychological phenomena such as memory, affect, or motivation. D. Dysfunctional schema develop over the course of life experience beginning in childhood.

D (According to Beck, schemas are relatively stable cognitive patterns on which attention to and interpretations of external events are based. In a depressed individual, the person's interpretation of external events is distorted to fit the schema. For example, a person who views him- or herself as worthless and deserving rejection will tend to interpret the statements and behaviors of others as rejection. Beck believed that life experiences, beginning in childhood, provide the basis for forming schemas. As an example, the belief that all losses are irreversible is a core schema that may have been formed due in large part to the death of a parent in childhood. Let's briefly go through the other choices. Choice A is incorrect because not all schemas -- even dysfunctional schemas -- cause problems. This is because, according to Beck, some schemas are inactive and remain so unless they are activated by external events. Choice B is wrong because Beck believed that schemas -- including core schemas, or basic assumptions about oneself and the world -- are readily available to the patient's conscious mind; this is one way that he distinguishes the cognitive therapy approach from psychoanalysis. And choice C is wrong because Beck believed that schemas are involved in memory, affect, and motivation, as well as all other functions related to perception and information processing.)

212. A 16-year-old girl tells her therapist that she wants to kill herself and that she's been thinking of using her mother's prescription pain pills to overdose. The therapist then tells her that he will have to tell her parents about her suicide risk. The girl becomes enraged and says that she would not have disclosed the information if she knew that the therapist would tell her parents. The therapist should: A. agree not to tell the parents if the girl agrees to a no-harm contract B. agree not to tell the parents, but tell them without the girl's knowledge C. tell the parents and refer the girl to another therapist because she will probably not be able to trust the therapist again D. tell the parents and apologize to the girl for having to do so

D (According to both ethics and law, a psychologist may disclose confidential information without the client's consent to protect the client from harm. This applies to minors and adults alike. In this case, it would most likely be appropriate to inform the girl's parents about her suicide risk. The reasons for this should be explained to the girl and the therapist should apologize for the breach of confidentiality. Given the apparent level of risk involved, it would probably not be sufficient to rely on a no-harm contract -A-. It would certainly be inappropriate and countertherapeutic to lie to the client -B-. And it may not be necessary to refer the client -C-. If handled empathically, it is likely that the girl will come to understand the therapist's reasons for disclosing to the parents and may come to appreciate that her safety was his overriding concern.)

122. A psychologist is contacted by email by a woman seeking online therapy to become more motivated to make some positive life changes. The psychologist agrees to provide the e-therapy and therefore should do which of the following in terms of charging this client for his services? A. have the client e-mail her credit card information to him before providing services B. charge per e-mail exchange, based on the fee that the client agrees to beforehand C. charge per minute, based on the fee that the client agrees to beforehand D. establish the fee with the client before providing his services

D (Although responses "B and "C are acceptable, only "D describes something that any e-therapist "shoulD do. One practice standard for psychologists offering services online is to state their fee structure clearly - clearly stating the amount of their fee on their Web page and communicating with clients about the fee before providing any services. Many e-therapists do what is described in answer "B - they charge their hourly fee and charge for the time they spend reading and answering each e-mail from a client. An e-therapist can have clients pay by mail, using a check or money order, or accept payment via a credit card. An e-therapist who accepts credit cards, however, must allow payment via a secure server or give the client a phone or fax number to call with his credit card information - he should never ask a client to send her credit card information in an e-mail -a.-, or via in any nonsecure Web form. Note: Additional practice standards for e-therapy include: Describing clearly and fully what services do and do not consist of; fully and accurately disclosing professional credentials, licensure and experience; fully inform clients about how online services work, the skills they need in order to use online services, reach an agreement on the frequency of service and what the possible benefits and risks are, including breaches of confidentiality; deliver services exactly as advertised; make provisions to ensure the confidentiality and privacy of online communications; maintain records of the services provided according to professional and state-mandated standards; and discuss the procedures to follow in an emergency. Legal issues related to e-therapy most often involve those having to do with regulation -i.e., licensure, malpractice coverage-, the therapist-client relationship, quality, and security and privacy. Given the unresolved legal issues associated with e-therapy, it considered by many to be too risky for psychologists to diagnose or treat mental disorders online. -See: Hsiung, R.C. -2002-. Suggested Principles of Professional Ethics for E-Therapy. E-Therapy: Case Studies, Guiding Principles, and the Clinical Potential of the Internet; And: International Society for Mental Health Online -ISMHO- & Psychiatric Society for Informatics -2000-. The Suggested Principles for the Online Provision of Mental Health Services. Accessed May 2007. http://www.ismho.org)

217. In a job selection decision, age can be used as an exclusionary criterion A. never. B. if there is a ruling from the appropriate federal agency allowing it. C. if there is a significant difference in mean ages of incumbents and applicants. D. if age is directly related to job performance.

D (Any limiting criterion is acceptable in a job selection procedure as long as the criterion is shown to be job-related. For instance, you could exclude people with very poor eyesight as air traffic controllers, you could exclude people with very little stamina and strength as firefighters, and so forth. Thus, if an employer can empirically demonstrate that being a certain age is a bona-fide occupational requirement, it could be used as a job criterion. Let's say you were hiring painters to work on the Golden Gate Bridge. You might very well find that people over age 60 couldn't do the job safely and well. If you could prove this finding empirically, you could use age as a criterion. Note, however, that if an employer is challenged on the use of a discriminatory exclusionary criterion, the employer bears the legal burden of demonstrating that it is job relevant.)

171. Early memory and attention deficits in Alzheimer's disease are believed to be caused by decreased activity in A. dopaminergic neurons. B. serotenergic neurons. C. GABAergic neurons. D. cholinergic neurons.

D (As knowledge about Alzheimer's accumulates, more is being learned about the neurotransmitters involved in the disease. Acetylcholine -a cholinergic neurotransmitter- was the first to be implicated and appears to be the neurotransmitter most involved, especially in early memory and attention deficits. However, serotonin, norepinephrine, and glutamate also seem to be involved but may be more important for symptoms that appear in the later stages of the disorder.)

159. Children raised by parents who are very demanding but warm are likely to be highly: A. neurotic B. oppositional C. insecure D. resilient

D (Authoritative parents are very demanding but also warm and responsive to their children. Children raised by authoritative parents tend be more mature and have better coping skills, or resilience to life stressors, than children raised with other parenting styles.)

135. Which of the following is NOT one of the levels of Kirkpatrick's training evaluation model? A. learning B. reaction C. results D. return on investment

D (Donald Kirkpatrick's -1959, 1998- model for evaluating training and learning programs includes four levels: reaction, learning, behavior and result. Each level of evaluation is used to assess the effectiveness of training programs and the effects of the training on the employees. Level 1 -b.- evaluations focus on the individual's response to the training or intervention. Level 2 -a.- measures what has been learned, often using pre/post-tests or end-of-course tests. Level 3 evaluations assess the impact of the intervention on the individual's behavior or performance in the workplace and level 4 -c.- measure the impact of training on the organization's results. -See: Kirkpatrick, D.L. -1998-. Evaluating Training Programs: The Four Levels -3rd ed.-. San Francisco, California: Berrett-Koehler.- Return on investment -ROI-, is a "level 5" evaluation, proposed by Jack Phillips, used to calculate the return on investment of an intervention or training. Phillips' Five Level ROI is an expansion of Kirkpatrick's model and encourages ROI evaluations for each of the first four levels. -See: Phillips, J.J. -2000-. Return on investment in training and performance improvement programs. Woburn, MA: Butterworth-Heinemann.)

153. Bilingual children ______________________ than monolingual children. A. perform no better or no worse academically B. perform worse academically C. have no better or no worse intellectual flexibility D. have increased intellectual flexibility

D (Evidence suggests that bilingual children perform better on tests of cognitive flexibility, divergent thinking, and metalinguistic awareness, than monolingual children. Findings on the academic performance of bilingual children are inconsistent and influenced by many factors such as the age of second language acquisition, community support, and academic services.)

120. A babysitter is watching her neighbor's 4-year old. The child wants some candy, but the babysitter was told by his mother not to give him any. After listening to the boy's whining for ten minutes, the babysitter gives in - she gives the boy some candy and he stops whining. In this situation, the babysitter's behavior -giving the child candy- is the result of A. positive punishment. B. negative punishment. C. positive reinforcement. D. negative reinforcement.

D (In this situation, the babysitter's "candy-giving behavior" has increased because it caused a stimulus to be removed -the child's whining-. This is negative reinforcement.)

168. In vivo exposure with response prevention involves: A. alternately presenting the CS and US. B. simultaneously presenting the CS and US. C. repeatedly presenting the US without the CS. D. repeatedly presenting the CS without the US.

D (In vivo exposure with response prevention involves exposing an individual to a feared or anxiety evoking stimulus and then blocking him or her from engaging in the usual avoidance response. The technique is based on the principle of classical extinction, which involves repeatedly presenting a conditioned stimulus -CS- without the unconditioned stimulus -US-. The idea is that the anxiety or fear developed through classical conditioning, or a pairing of a conditioned stimulus -the feared stimulus- and an unconditioned stimulus -a stimulus that naturally causes fear-.)

110. Which of the following is NOT one of Baumgartner's incorporation phases of an HIV/AIDS diagnosis into identity? A. post-immersion turning point B. post-diagnosis turning point C. disclosure D. stabilization

D (Informed by narrative and transformational learning approaches to identity development, Lisa Baumgartner's study -2001- on the incorporation of an HIV/AIDS diagnosis into one's identity identified a six-phase process including: diagnosis, post-diagnosis turning point, immersion, post-immersion turning point, integration and disclosure. The diagnosis phase involved the common reactions of shock, fear, denial, and relief. Post-diagnosis turning point -b.- was associated with social interaction or a "catalyst experience." In the immersion phase, people became immersed in the HIV/AIDS community; the HIV/AIDS identity was central and they educated others. Whereas the post-immersion turning point phase caused a revaluation of priorities. The integration phase was associated with decentralizing -internal experience of the HIV/AIDS identity- and balancing, such as engaging in activities unrelated to HIV/AIDS. Disclosure was woven throughout the process however initially individuals only told significant others, then made public disclosures, and finally made situational disclosures on a need-to-know basis. -See: Baumgartner, L. M. -2002- Living and Learning with HIV/AIDS: Transformational Tales Continued. Adult Education Quarterly, Vol. 53, No. 1, 44-59.)

145. Reduced incidence of substance abuse among Native American adolescents would most likely result from: A. normalizing substance use within tribal communities B. increasing the legal consequences of substance use C. aversive conditioning D. increasing bicultural competence

D (Interventions designed to increase "bicultural competence" have been found effective in preventing substance abuse among Native American adolescents. These interventions help individuals develop coping skills that "blend the adaptive values and roles of both the culture in which they were raised and the culture by which they are surroundeD -LaFromboise, Assertion training with American Indians: Cultural/behavioral issues for trainers. Las Cruces: New Mexico State University, 1982, p. 12-. Normalizing substance use -A- would likely increase the incidence of substance abuse and has been regarded as one of the factors that contributes to the high rate of substance abuse among Native Americans. Increasing legal consequences of substance use has not been found to be a very effective deterrent for adolescents presumambly because adolescents do not always make rational decisions based on consideration of the perceived consequences of committing a crime. -D. M. Kahan, Between economics and sociology: The new path of deterrence. Michigan Law Review, 1997, 95, 2477-2497-. Finally, aversive conditioning -C- is more likely to be used as a treatment, rather than a method of prevention.)

182. Which of the following would be used to determine the probability that examinees of different ability levels are able to answer a particular test item correctly? A. criterion-related validity coefficient B. item discrimination index C. item difficulty index D. item characteristic curve

D (Item characteristic curves -ICCs-, which are associated with item response theory, are graphs that depict individual test items in terms of the percentage of individuals in different ability groups who answered the item correctly. For example, an ICC for an individual test item might show that 80% of people in the highest ability group, 40% of people in the middle ability group, and 5% of people in the lowest ability group answered the item correctly. Although costly to derive, ICCs provide much information about individual test items, including their difficulty, discriminability, and probability that the item will be guessed correctly.)

207. A family therapist who uses anecdotes from her own family to emphasize similarities with the client family is doing which of the following? A. reframing B. triangulating C. diffusing boundaries D. joining

D (Joining, a technique used by structural family therapists, involves taking steps to become a family insider by, among other things, assuming the same type of communication style, sharing personal stories, etc.)

125. Kubler-Ross proposed people go through which of the following stages when faced with their own death: A. denial, anger, depression, acceptance and hope B. denial, anger, bargaining, acceptance and hope C. denial, anger, bargaining, depression and hope D. denial, anger, bargaining, depression and acceptance

D (Kubler-Ross -1969- proposed five stages: denial, anger, bargaining, depression and acceptance -DABDA-. Research indicates that these feelings don't necessarily occur in a fixed sequential order. Although the feeling of hope was not identified by Kubler-Ross, it is a common emotion in the terminally ill.)

56. The cortex is the least developed part of the brain at birth. Subsequent development is due primarily to: A. the growth of new neurons. B. the growth of new neurons and glial cells. C. increases in the size of existing neurons. D. myelination of existing neurons.

D (Nearly all of the neurons are present at birth, and continued development of the brain following birth is due primarily to an increased number of dendrites and myelination of the existing neurons.)

203. The four main stages of neural development are: A. meiosis, mitosis, migration, myelination B. mitosis, meiosis, differentiation, myelination C. differentiation, mitosis, migration, myelination D. proliferation, migration, differentiation, myelination

D (Neural development involves the stages of proliferation, migration, differentiation, and myelination. The proliferation stage includes the production of cells. Migration is the second stage of development during which the cells move or migrate to its ultimate destination in the nervous system. Following migration is differentiation, in which cells develop the unique characteristics of nerve cells. During the fourth stage, myelination, the axons of some cells become surrounded -insulated- by glial cells. Meiosis is the process of cell division that produces cells with one-half the number of chromosomes as the parent cell. Mitosis is the process of cell division that produces cells with the same number of chromosomes as the parent cell.)

138. Studies investigating the relationship between suicide and health status in the elderly suggest that: A. contrary to what is commonly believed, there is no clear relationship between these two variables. B. physical health is a predictor of suicide in the elderly only when it is combined with social isolation. C. physical health is a predictor of suicide in the elderly only for males. D. concern about physical health is one of the most common reasons for suicide in the elderly.

D (Older people have more health problems than younger ones and are often faced with a terminal illness, chronic pain, and/or an inability to take part in normal activities because of the illness. Not surprisingly, physical illness has been found in several studies to be the most common reason for suicide among the elderly.)

220. Side effects of propranolol -Inderol- are most likely to include: A. tachycardia and mania B. tachycardia and tremor C. bradycardia and hypersomnia D. bradycardia and depression

D (Propranolol -Inderal- is a beta-blocker often used to treat hypertension, migraine, essential tremors and is also useful in reducing the physical symptoms of anxiety. Side effects of Inderal include bradycardia -slow heart rate-, hypotension, fatigue, sexual dysfunction, and depression. )

115. A psychologist in a rural area is referred a 17-year-old male who has been abusing OxyContin. Although the psychologist is trained and experienced in the treatment of substance abuse, she has not received training in, nor experience with, OxyContin abuse. The psychologist should: A. refer the patient to a psychologist outside the area who is experienced in the treatment of OxyContin abuse B. agree to treat the patient after attending a workshop on the treatment of OxyContin abuse C. refer the patient for inpatient detoxification D. provide the patient with treatment and read relevant literature

D (Psychologists have an ethical responsibility to practice within the boundaries of their competence. However, according to APA's Ethics Standard 2.01-d-, "When psychologists are asked to provide services to individuals for whom appropriate mental health services are not available and for which psychologists have not obtained the competence necessary, psychologists with closely related prior training or experience may provide such services in order to ensure that services are not denied if they make a reasonable effort to obtain the competence required by using relevant research, training, consultation, or study." It should not, therefore, be necessary to refer the patient to a provider out of the area. It also would not be in the patient's best interest to postpone treatment until the psychologist can attend a workshop on the subject -B-. And, although OxyContin abuse is serious and potentially fatal, it generally would not require inpatient detoxification -C- unless the abuse progressed to dependence.)

211. What is the least helpful suggestion you can give to parents when they are going through a divorce or marital conflict? A. avoid exposing the children to any conflicts B. allow the children to participate in decisions regarding visitation rights C. in the long run, it is better for the children if you divorce than if you remain in an unhappy marriage D. remarry as soon as possible

D (Research has identified a number of factors associated with poorer adjustment in children whose parents divorce. For instance, rapid remarriage of one or both parents is predictive of poorer adjustment. The other suggestions are likely to be helpful -or at worst, benign-: exposure to parental conflict, parental continuance of an unhappy marriage, and failure to allow children to establish a relationship with the noncustodial parent are all factors associated with poorer long-term adjustment in children of divorce.)

134. The most correct statement that can be made about biofeedback as a therapeutic treatment for pain reduction is that A. the significant effects found can be accounted for by the type of instrumentation employed. B. it is effective for certain types of patients only. C. its effectiveness has diminished over the years. D. it is probably no different from relaxation training in its effectiveness.

D (Research on the efficacy of biofeedback for pain reduction indicates that it is as effective as treatments such as relaxation training. This is true for the treatment of most disorders or conditions, however in cases such as the treatment of Raynaud's Disease or migraine headaches, some studies suggest that biofeedback is the treatment of choice.)

201. Most studies on child sexual abuse have not found which of the following to be associated with more severe outcomes? A. abuse by family member B. onset of abuse at an earlier age C. abuse by a much older perpetrator D. male gender

D (Research results on the psychological and behavioral effects of child sexual abuse have been inconsistent however most studies indicate no consistent difference in outcomes for male and female victims of sexual abuse. In studies that found a gender difference, the outcomes for females was worse than for males. Worse outcomes are associated with abuse by a family member -a.-, onset of abuse at an earlier age -b.- and abuse by a much older perpetrator. -See: National Research Council, Understanding Child Abuse and Neglect, Washington, DC, National Academy Press, 1993.)

199. According to research on parenting styles, which of the following describes the parents who are most likely to raise very aggressive children? A. attentive parents who are very controlling of their children's behavior B. parents who use frequent and intermittent violence and are very controlling of their children's behavior C. loving parents with a laissez-faire attitude toward their children's behavior D. parents who use frequent and intermittent violence and have a laissez-faire attitude toward their children's behavior

D (Researchers in the 1950s identified two dimensions of parenting styles: permissiveness and affection. Permissiveness is a continuum that ranges from autonomy on one extreme to control at the other; affection is a continuum that ranges from hostility to love. Parents who combine hostility with autonomy -e.g., those who combine violent discipline with a laissez-faire attitude toward their children- are likely to produce disobedient and aggressive children.)

118. An organization is considering different selection techniques and comparing the respective predictive validity of each. The validity of structured interviews as predictors of performance: A. are the most valid across different jobs B. are no more valid than unstructured interviews when used alone C. is increased when the interview includes both structured and unstructured items D. is increased when the interview is used in combination with a measure of general mental ability

D (Summarizing the predictive validity of interviews and other selection techniques in a meta-analysis of the research, Schmidt and Hunter -1998- report measures of general mental ability are the most valid predictors across different jobs -a.-. For structured and unstructured interviews, they note corrected validity coefficients of .51 and .38, respectively -b.-. When an interview is combined with another selection procedure, especially a general mental ability test, the predictive validity of interviews is increased [The validity and utility of selection measures in personnel psychology: Practical and theoretical implications of 85 years of research findings, Psychological Bulletin, 124-2-, 262-274)

177. The probands, in a study comparing characteristics of adult ADHD patients, with characteristics of their first degree and second degree biological relatives and non-patients -controls-, are: A. non-patients B. first degree relatives C. first and second degree relatives D. ADHD patients

D (The ADHD patients are the probands in this study. Probands, or index cases, are the individuals who are first brought to the attention of the researcher - i.e., individuals manifesting the characteristic of interest or disease.)

67. In regard to substance use, the Americans with Disabilities Act A. protects alcoholics and drug addicts as "qualified individuals" with a disability. B. prohibits administering a drug test to an individual who is participating in a drug rehabilitation program. C. prohibits drug testing from being administered to a prospective employee after a job offer is made. D. protects individuals participating in a drug rehabilitation program who are no longer using drugs.

D (The Americans With Disabilities Act -ADA- does not consider individuals who are "currently engaging in the illegal use of drugs" as suffering from a disability on that basis. In other words, it does not protect current drug users. However, it does protect past substance addicts -- as long as the individual is currently participating in or has completed a supervised rehabilitation program, and the person is not currently using drugs. In regard to drug testing, the act does not identify any circumstances in which drug testing is prohibited. In fact, even though it protects former substance addicts, the Act permits employers to use "reasonable means" -- including drug tests -- to verify that such persons are no longer using substances.)

142. You are working as a school psychologist and you meet with a couple who wishes to inspect a copy of their child's complete school records. In this situation, which of the following statements is most applicable? A. You should deny the parents' request, since these records are confidential. B. You should tell the parents that you cannot release the records without the authorization of the school principal. C. You should present a summary of the records, since the parents will not be qualified to interpret the complete records. D. Under federal law, parents have the right to inspect the complete records, and, if they wish, challenge their contents.

D (The Buckley Amendment gives parents the right to inspect their child's school records and to challenge their contents. Of course, in this situation, you should provide a summary if necessary and make yourself available for a discussion of the records' contents. However, if the parents insist on seeing the complete records, they have that right.)

146. Atrophy of the caudate nucleus is found in: A. Obsessive-Compulsive Disorder B. Jacob Creutzfeld Disease C. Schizophrenia D. Huntington's Disease

D (The caudate nucleus is part of the basal ganglia which is responsible for initiating movement. Atrophy of the caudate nucleus is found in patients with Huntington's Disease which is a disorder which includes affective, motor, and cognitive symptoms. Obsessive Compulsive Disorder -"A- is believed to be related to an overactive caudate nucleus.)

184. "According to current research, the Rorschach has very low validity." This statement is A. true. B. true, except for children between ages 5-10. C. true, except for its use with a nonverbal population. D. false.

D (The correct answer is false. The Rorschach, while having a reputation for low validity due to its clinical origins, has increased in validity since the development of John Exner's scoring system. Hiller, Bornstein, Rosenthal, Berry and Brunell-Neuleib report in Psychological Assessment -September, 1999- that since earlier critiques, two recent developments have given the Rorschach a more favorable outlook. First is the development of Exner's Comprehensive System for the Rorschach -Exner, 1974; 1978- which systematized scoring procedures for the Rorschach, and reported findings from a large archive of Rorschach protocols of normal adults, as well as from various psychiatric groups. Second, two meta-analyses were published, both comparing criterion-related validity evidence for the Rorschach to that of its chief rival, the MMPI -Atkinson, 1986; Parker, Hanson, and Hemsley, 1989-. These studies conclude that validity is roughly equivalent for both instruments. Moreover, in Hiller et al.'s -1999- article, "A Comparative Meta-Analysis of Rorschach and MMPI Validity," their findings indicate that the MMPI had larger validity coefficients than the Rorschach for studies using psychiatric diagnoses and self-report measures as criterion variables, whereas the Rorschach had larger validity coefficients than the MMPI for studies using objective criterion variables.)

209. You are a psychologist in private practice and, due to a heavy case load, you decide to work with another psychologist. The second psychologist will see many of the clients and share office space with you. You agree to give this psychologist 50% of all fees received. This arrangement is A. unethical, because the ethical standards prohibit fees for referrals. B. unethical, because it threatens the best interests of the clients. C. unethical, because the clients will be misled about their fees. D. ethical.

D (There is nothing inherently unethical about this arrangement, and it is not barred by APA's ethical standards. The standards do prohibit payment for referrals that are based on the referral itself and not based on services rendered. However, both psychologists in this case will apparently be rendering services, so the arrangement appears acceptable.)

4. A factorial design, unlike a two group design: A. allows more independent variables to be studied B. requires a larger sample C. shows the effect of an independent variable on the dependent variable D. cannot detect a curvilinear relationship between variables

A ( In a two group design, one group is exposed to a treatment and another, control group, is not exposed or gets a different treatment. The results of both groups are tested in order to compare the effects of treatment. A factorial design is a design with more than one independent variable. In this design, the independent variables are simultaneously investigated to determine the independent and interactive influence they have on the dependent variable. The effect of each independent variable on the dependent variable -c.- is called a main effect and in a factorial design there are as many main effects as there are independent variables. An interaction effect between two or more independent variables occurs when the effect that one independent variable has on the dependent variable depends on the level of the other independent variable. At least three levels must be used to predict a curvilinear relationship.)

63. Of the following, which is most frequently used to detect brain damage in the early stages of emergency room -ER- treatment? A. CT B. EEG C. PET D. MRI

A ( The CAT -Computerized Axial Tomography- or CT scan, and the Magnetic Resonance Imaging -MRI- take a series of images at different levels of the brain giving information and a direct visualization of structures and features. The MRI does this with magnetic fields; the CAT scan uses x-rays. The CAT scan is superior to the MRI in detecting fresh blood in and around the brain and often is repeated to ensure that a brain injury is not becoming more extensive, usually in the early stages of ER treatment. Another difference is CT scans cost substantially less than MRIs. In general, the MRI -d.- provides more detail than the CAT scan and can be used in examining the central nervous system, as well as, used to identify tumors, strokes, degenerative diseases, inflammation, infection, and other abnormalities in organs and other soft tissue of the body. A procedure that uses electrodes on the scalp to record electrical activity of the brain is an Electroencephalogram -EEG- -b.-. It is used for detection of epilepsy, coma, and brain death. A PET -Positron Emission Tomography- scan measures metabolic processes, thus allowing an appraisal of how the brain is functioning. It tracks natural compounds, such as glucose, as the brain metabolizes them. By showing the areas of different metabolic activity, it then becomes easier to make diagnoses, such as determining the areas responsible for epileptic seizures. )

21. First degree relatives of schizophrenics are more likely is to be diagnosed as: A. Schizotypal B. Schizopheniform C. Schizoaffective D. Borderline Personality disorder

A (According to DSM-IV, Schizotypal Personality Disorder appears to aggregate familially and is more prevalent among the first-degree biological relatives of individuals with Schizophrenia than among the general population.)

31. A man comes in for therapy very upset because he has had problems achieving and maintaining an erection. He has seen advertisements for Viagra in the Men's Journal and feels this would be the solution for him. What type of disorder does he have? A. Male erectile disorder B. Premature ejaculation C. Psychopharmacological consumerism D. Sexual aversion disorder

A (According to DSM-IV, the criteria for a diagnosis of male erectile disorder is: persistent or recurrent inability to attain or maintain an adequate erection, AND the disturbance causes marked distress or interpersonal difficulty. anger have been found to be the primary risk factors.)

10. A 16-year old girl is sexually promiscuous and does not practice safe sex. When confronted with information about sexually transmitted diseases and pregnancy, she states "that could never happen to me." The girl's behavior is characteristic of A. adolescent egocentrism. B. concrete operational thought. C. teenagers who have been sexually abused. D. an immature system of defense mechanisms.

A (According to Piaget and researchers who have studied his theories, adolescents are prone to what has been termed "formal operational egocentrism" or "adolescent egocentrism." This is characterized by a number of beliefs and modes of thinking, including the belief that the world can only become a better place through implementation of a grand idealistic system, the belief that others are as concerned with the adolescent's behavior as the adolescent him- or herself is, and -as in this question- a strong faith in one's own invulnerability and uniqueness.)

12. A sequence of behaviors, in which each behavior serves as reinforcement for the previous behavior, is the outcome of A. chaining. B. shaping. C. positive reinforcement. D. graded exposure.

A (According to the operant conditioning paradigm, chaining is how complex behaviors made up of a sequence of simpler behaviors are developed. In chaining, each response serves as reinforcement for the previous behavior and a discriminant stimulus for the next behavior on the chain.)

32. In addition to improved interpersonal relationships, a primary goal of interpersonal therapy -ITP- is: A. alleviating or reducing symptoms. B. satisfying needs in responsible ways. C. resolving underlying conflicts. D. living in an authentic, meaningful way.

A (Based on a medical model, interpersonal therapy -ITP- views mental disorders as illnesses and focuses on symptom reduction in addition to improving interpersonal relationships.)

39. A loss of memory for autobiographical information is referred to as: A. functional amnesia B. anterograde amnesia C. retrograde amnesia D. malingering

A (Functional amnesia is a condition, caused by a psychological trauma, in which individuals are unable to remember significant events in their lives, i.e., autobiographical information. Anterograde amnesia -B- is an impaired ability to form new permanent memories. Retrograde amnesia -C- is an inability to recall previous memories -i.e., for events that occurred prior to a head trauma-, and is not necessarily for, or limited to, autobiographical information. Someone who is malingering -D- could feign memory loss for autobiographical memories; however, most cases of autobiographical memory loss are not due to malingering.)

8. A moderator is A. a variable that affects the direction or strength of the association between two other variables. B. an explanation of how external physical events take on internal psychological significance. C. a variable that identifies the relationship between two variables and serves to magnify the strength of the variables. D. a variable that accounts for the relationship between two variables.

A (In general, a moderator is a qualitative -e.g., race, sex, class- or quantitative -e.g., level of reward- variable that affects the direction and/or strength of the relation between an independent or predictor variable and a dependent or criterion variable. A moderator only influences the strength of the relationship between two other variables, it doesn't fully account for it. In contrast, a variable functions as a mediator to the extent that it accounts for the relation between the predictor and the criterion.)

83. Learning disabilities are defined by discrepancies between A. achievement and aptitude. B. full scale IQ score and overall intelligence. C. verbal and performance IQ scores. D. academic skills and adaptive coping skills.

A (In the words of the DSM-IV, "Learning Disorders are diagnosed when the individual's achievement on individually standardized tests in reading, mathematics, or written expression is substantially below that expected for age, schooling, and level of intelligence." In other words, they involve a discrepancy between what the person has achieved and what would be expected given the person's potential to achieve, or aptitude.)

68. The difference between insanity and psychosis is that: A. insanity is a legal term and psychosis is a term used in mental health literature B. insanity is an outdated term and is no longer applied in legal cases, while psychosis is used both in mental health and legal literature C. insanity is legally a broader term, and subsumes the term psychosis D. insanity refers to disturbances in thought and emotion, while a psychotic disturbance, by definition, affects thought only

A (Insanity is strictly a legal term; it usually means that a person is in such a mental state that he or she cannot distinguish between right or wrong. Psychosis, on the other hand, is a psychiatric term.)

38. The initial stage in structural family therapy is to "join" the family system. To do so, which of the following would be most useful? A. mimesis B. enactment C. positioning D. alignment

A (Mimesis is the term used to describe adopting a family's communication and affective style and is one way of joining the family system.)

45. The WISC-III subtests that are most sensitive to perceptual-motor problems are: A. block design and object assembly. B. object assembly and picture completion. C. digit span and coding. D. picture arrangement and picture completion.

A (Of the choices listed, the block design and object assembly subtests are the most direct measures of perceptual and motor skills.)

52. A member of which of the following groups is most likely to be admitted to an inpatient mental health facility? A. never married men B. divorced/separated men C. married men D. never married women

A (Of the groups listed, never married men have the highest rates of admission to inpatient mental health facilities regardless of the type of facility -state/county versus private-. The other groups listed have similar rates; but, if you have to choose the group with the lowest rate, the best answer is never married women -answer D-.)

15. According to Jerry Patterson and associates the families of highly aggressive boys are characterized by: A. coercive interactions and poor parental monitoring B. Coercive interactions and high parental monitoring C. Low interaction and low parental monitoring D. Excessive television viewing

A (Patterson, Chamberlin and Reid 1982 "A Comparative evaluation of a parent-training program" Behavior Therapy, 13-5- 638-650 proposed a coercive family interaction model. It hypothesized that children first learn aggressive behaviors from their parents. These parents also use harsh discipline and reward their children's aggressiveness with attention and approval. As a result, the aggressiveness of the parent-child interaction continues to escalate.)

46. According to Self Verification Theory, a person who believes that he dances poorly would prefer to be told by friends: A. "You are a pretty bad dancer" B. "You are not a bad dancer" C. "You should become a professional dancer" D. nothing about dancing

A (Self Verification theory proposes that people need and seek confirmation of their self-concept, regardless of whether their self-concept is positive or negative. Thus, people prefer to be right rather than happy. According to this theory, a person who dances poorly would prefer to be told so -assuming the other's evaluation matches the person's self evaluation-.)

93. Latane called this a social disease. It occurred when high-level employees were assessed on a combined effort. It turned out they produced less than when they were working individually. It's called: A. Social loafing B. Group think C. Social polarization D. Social facilitation

A (Social loafing or Latane's "social disease" is the discovery that in regard to work, individual output declines when people are working together as a group. However, social loafing does not occur under all conditions. It is reduced or eliminated when participants believe that their individual contributions are identifiable or uniquely necessary for the group to succeed. -B- "groupthink" is the tendency for a group to make an irrational or impulsive decision in order to reach consensus. -C- "Group polarization" refers to the tendency of individuals who start with a similar view to end with a more extreme position after group discussion.)

22. A psychologist is undertaking a research program in an elementary school on the effect of a new counseling program on enhancing children's self-esteem. The psychologist has received permission from the school's principal, the teachers, and from the parents of each student. However, when she explained the program to the children, some of them objected to being part of the study. The psychologist should A. not use these children in her study. B. call the parents of these children and ask them to try to convince the children to participate. C. use the children in her study because she has all necessary legal consents. D. re-design the study to eliminate the objection the children have to participating.

A (The best idea is to just leave out these children. Participants have the right to decline to participate in research, even if they are children. It wouldn't be practical or prudent to ask her to re-design the whole experiment because a handful of children object to being participants. Thus, the best answer is to just do the study without using the children who object to participating.)

2. The findings of the National Institute of Mental Health -NIMH- research project regarding the relative effectiveness of cognitive-behavioral therapy and antidepressant medication in the treatment of depression indicate that A. both treatments are about equally effective. B. cognitive/behavioral therapy is more effective in the short-term, while medication is more effective in the long-term. C. cognitive-behavioral therapy has an immediate effect, while medication has a delayed effect. D. cognitive-behavioral therapy is more effective.

A (The findings of the NIMH research project were that, on an overall level, cognitive-behavioral therapy -CBT- and antidepressant medication are about equally effective in the treatment of depression. There was some evidence that the treatments differed in the types of depression for which they are most effective. Specifically, some evidence suggested that CBT is more effective for milder forms of depression, while medication is more effective for relatively severe forms. But in terms of overall effectiveness, both treatments were found to be about equal.)

70. When undertaking token economies with seriously disturbed individuals in mental institutions, one of the major problems with the program's efficacy has to do with A. generalization of behaviors. B. choice of reinforcers. C. exchange ratio. D. reinforcement value.

A (Use of a token economy involves administering secondary reinforcers such as a token each time the person engages in a desired behavior, or taking away a reinforcer when a person engages in an undesired behavior. The tokens can then be exchanged for primary reinforcers such as food or desired activities. Token economies are commonly used in institutional settings. A problem with them is that behaviors learned often fail to generalize to the real world, since tokens are not available in the real world every time we do something right.)

43. Research on psychotherapy outcomes suggest that, overall, culturally diverse groups do about as well as Anglo clients. However, as a specific group, less favorable outcomes are shown by A. African-Americans. B. Asian-Americans. C. Hispanics. D. Anglo-Americans.

A (Using a Global Assessment Scale, Sue -1991- found that patients from all groups show improvement in scores following therapy, but as a group, African-Americans have less favorable outcomes than Asian, Hispanic, or Anglo Americans.)

28. Repetitive exercise and hypnosis are therapeutic techniques used to establish the "relaxation response" which involves ____________________________ activity. A. decreased parasympathetic nervous system B. decreased sympathetic nervous system C. increased somatic nervous system D. increased autonomic nervous system

B ( The relaxation response is, in effect, the opposite of the "fight or flight" response to stressful or threatening situations which over time may produce hypertension, cardiac and other problems that may seriously affect health. Herbert Benson and his colleagues -1971- studied the affects of meditation on people with high blood pressure brought on by the everyday stress of living and described a physiological response that decreased sympathetic nervous system activity resulting in decreased metabolism, decreased heart rate, decreased blood pressure, and decreased rate of breathing, as well as slower brain waves. This reaction was coined the "relaxation response." Further findings indicate relaxing just 20 minutes each day can be beneficial to both physical and mental health. -See: Benson, H. -2000-. The relaxation response - updated and expanded -25th anniversary edition-. New York: Avon.)

36. During the 10th psychotherapy session with a client, you realize that your client has recently started dating your best friend. You should: A. refer the client to another therapist B. discuss this with your client immediately C. discontinue your relationship with your friend D. wait and address the problem with your client if a conflict becomes imminent

B (Although the Ethics Code does not specify exactly how to resolve this type of ethical dilemma, it does state, "If a psychologist finds that, due to unforeseen factors, a potentially harmful multiple relationship has arisen, the psychologist takes reasonable steps to resolve it with due regard for the best interests of the affected person and maximal compliance with the Ethics Code" -Ethical Standard 3.05b-. Consistent with the intent of the Code, the best option would probably be to discuss the matter with your client immediately. After discussing it with your client, additional steps, such as referring the client, may be appropriate. )

87. According to Piaget, when a child accommodates new information by forming a new schema or modifying an existing one, this results in A. decentration. B. equilibration. C. assimilation. D. symbolic representation.

B (As defined by Piaget, equilibration is a state of cognitive balance. The need for balance is what motivates the individual to assimilate and accommodate new information.)

100. A child has been assigned a new legal guardian. His biological parents had a psychological evaluation completed and had received the results. The new legal guardian is requesting a copy of the report. What is the most correct thing to do? A. Get a signed release from the biological parents B. Provide the legal guardian with a copy of the report C. Send the report to the court mediator D. Keep the evaluation on file until the child is emancipated; then forward to him

B (As legal guardian he or she has the right to a copy of the report. The key term is "legal". A court has determined legal access in this case so you as the psychologist only need to be guided by the law.)

11. Communication-interaction therapy espouses that communication has both a "report" function and a A. Principle of equifinality B. Command function C. Circular model of causality D. Paradoxical strategy

B (Family therapists from the Mental Research Institute in Palo Alto such as Gregory Bateson, Virginia Satir, and Jay Haley described communication as having a "report function" that contains the content or informational aspect of the communication, and the "command function", that is often conveyed nonverbally and exemplifies the relationship between the communicators. The other choices are other concepts from the Mental Research Institute. "Principles of equifinality" refers to the idea that no matter where the system change occurs, the end result is the same. "Circular model of causality" is a concept in their approach that describes a symptom as both a cause and an effect of dysfunctional communication patterns. "Paradoxical strategies" include prescribing the symptom and relabeling, or changing the label a family attaches to a problem in order to change the meaning.)

14. Reactive Attachment Disorder, Inhibited Type involves A. indiscriminate attachments. B. hypervigilance. C. over-attachment to a primary caretaker and fear of others. D. symptoms of Posttraumatic Stress Disorder in children.

B (Reactive Attachment Disorder occurs in children below the age of 5 and is characterized by disturbed social relatedness. There are two subtypes: inhibited and disinhibited type. This question is about the former, which involves inhibition, hypervigilance, and ambivalent responses in social situations. The other subtype, disinhibited type, involves indiscriminate attachments and sociability -e.g., familiarity with strangers, lack of selectivity in choice of attachment figures-. In both subtypes, evidence of pathogenic care -e.g., disregard for the child's basic physical or emotional needs- must be present before the diagnosis can be made.)

77. Which of the following antidepressants is associated with the treatment of attention-deficit/hyperactivity disorder -ADHD-, enuresis, and decreasing the desire to binge and purge? A. sertraline B. imipramine C. paroxetine D. fluvoxamine

B (Research has shown tricyclic antidepressants -TCA- to be effective in the treatment of anxiety disorders, refractory pain syndromes, reducing binging and purging episodes in bulimia nervosa, and as alternatives to the stimulants in the treatment of attention-deficit/hyperactivity disorder. Furthermore, the TCA, imipramine, is the drug of choice in the treatment of nocturnal enuresis and for the treatment of cataleptic episodes associated with narcolepsy, along with the TCA clomipramine. Sertraline, paroxetine and fluvoxamine are selective serotonin reuptake inhibitors -SSRI-. SSRIs are also effective in the treatment of anxiety disorders, reducing binging and purging episodes, and some types of chronic pain.)

27. Which of the following is either a symptom of or a requirement for the DSM-IV diagnosis of Separation Anxiety Disorder? A. onset before the age of 2 1/2 B. fear that the attachment figure will die C. symptoms do not persist after the age of 18 D. need for constant attention from the primary attachment figure

B (Separation Anxiety Disorder involves developmentally inappropriate and excessive anxiety concerning separation from home or from those to whom the individual is attached. For a diagnosis to be made, three signs or symptoms must be present, one of which is excessive worry that harm will befall a major attachment figure. Thus, B is the best answer. Contrary to choice A, the onset of the disorder need not be before the age of 2 1/2; in fact, separation anxiety in infancy is considered to be developmentally appropriate. Choice C may have been more difficult to eliminate: Although a diagnosis of Separation Anxiety Disorder requires an onset before the age of 18, symptoms may persist after the person turns 18. An excessive need for attention -choice D- is a commonly associated feature of this disorder, but it is not a core symptom.)

9. Which of the following illustrates the concept of shrinkage? A. extremely depressed individuals obtain a high score on a depression inventory the first time they take it, but obtain a slightly lower score the second time they take it B. items that have collectively been shown to be a valid way to diagnose a sample of individuals as depressed prove to be less valid when used for a different sample C. the self-esteem of depressed individuals shrinks when they are faced with very difficult tasks D. abilities such as short-term memory and response speed diminish as we get older

B (Shrinkage can be an issue when a predictor test is developed by testing out a pool of items on a validation -"try-out"- sample and then choosing the items that have the highest correlation with the criterion. When the chosen items are administered to a second sample, they usually don't work quite as well -- in other words, the validity coefficient shrinks. This occurs because of chance factors operating in the original validation sample that are not present in the second sample.)

72. The California Tarasoff Statute: A. Changed the Ethics Code in Canada and the United States B. Extended the duty to warn clause C. Was necessary as issues implicated by the Tarasoff case account for a majority of forensic cases D. Applies to everyone working in a public setting

B (The California Tarasoff statute gives clinicians explicit guidance about when a duty to act arises and tells clinicians what actions fulfill their duty. The first Tarasoff decision established a "duty to warn," however, the case was reheard several years later and the Tarasoff II decision modified the duty to warn to a "duty to protect." Choice C contradicts the conclusion by Behnke, Preis and Bates in "The Essentials of California Mental Health Law" Norton and Company, 1998, pg. 9 in which they state, "issues implicated by the Tarasoff case and its legal progeny account for a small percentage of forensic cases.")

23. Which of the following is NOT true regarding the Halstead-Reitan Neuropsychological Test Battery? A. It provides information on a range of cognitive strengths and weaknesses. B. It can usually be completed within 45 to 60 minutes. C. It must be administered by a highly trained examiner. D. The results are reflected in a combined score known as the Impairment Index.

B (The Halstead-Reitan Neuropsychological Test Battery is a set of tests designed to assess attention, language, memory, abstract thinking, motor speed, and spatial reasoning -A-. The purpose of the test is to provide an overall assessment of brain function. The battery must be administered by a highly trained examiner -C-. The results of each subtest are combined into an Impairment Index -D-. However, contrary to choice B, the battery usually requires 4 to 5 hours to administer.)

80. The best way to make an accurate and reliable diagnosis of Alzheimer's Disease is by way of A. neuroimaging techniques -e.g., CT scan, MRI-. B. a brain autopsy. C. ongoing observation of the patient. D. mental status exam.

B (The key words in this question are "accurate and reliable." There is no laboratory test available which confirms definitively Alzheimer's Disease. Diagnosis of the disorder involves confirming that the diagnostic criteria are met and ruling out other possible causes of the Dementia. Diagnosis is aided by the use of neuroimaging techniques, such as computerized tomography -CT-, magnetic resonance imaging -MRI-, and positive-emission tomography -PET-. Indeed, properly applied diagnostic techniques result in an 85% accuracy rate in diagnosing the disorder. However, at this time, the only way to absolutely confirm the diagnosis is via brain autopsy or biopsy.)

42. The appropriate kind of validity for a test depends on the test's purpose. For example, for the psychology licensing exam: A. construct validity is most important because it measures the hypothetical trait of "competence." B. content validity is most important because it measures knowledge of various content domains in the field of psychology. C. criterion-related validity is most important because it predicts which psychologists will and will not do well as professionals. D. no evidence of validity is required.

B (The psychology licensing exam is considered a measure of knowledge of various areas in the field of psychology and, therefore, is essentially an achievement-type test. Measures of content knowledge should have adequate content validity.)

61. A reason why the rational-economic model of decision-making usually does not prove viable in an organization is that A. it allows for the consideration of a limited number of options. B. it places too many demands on the individual and the organization. C. it arouses resentment in organization members who are excluded from the decision-making process. D. humans are irrational and emotional by their nature.

B (The rational-economic model assumes that decision-making involves a rational process in which all alternatives are considered and the best possible decision is eventually made. The model requires that decision-makers obtain information about and consider all possible alternatives before making a decision. This requirement places an inordinate demand on the people involved. Thus, in the real world, decisions are usually made based on the information that is available given restraints of resources such as time, money, and personnel.)

88. An individual who produces normal sounding speech that makes little or no sense and is usually unaware of this deficit is most likely to have damage in the: A. cingulate sulcus B. temporal lobe C. corpus callosum D. frontal lobe

B (This is a description of Wernicke's aphasia, also known as receptive or fluent aphasia, which can be caused by damage to Wernicke's area in the temporal lobe.)

3. You are a small-town psychologist in a rural area and a client comes in and states that she is depressed and is having headaches. She has babysat with your child several times over the last year. She would like you to give her biofeedback sessions. You should first: A. Complete the biofeedback treatment and not ask her to babysit in the future. B. Refer her to her family physician C. Refer her to a psychologist in another town for therapy; it will be OK for you to complete the biofeedback sessions D. Refer her to a biofeedback specialist in another town

B (This question is more about an awareness of your limitations than it is about a dual relationship. The first step, whether you decide to take the case or not, should be to refer her to her family physician to rule out medical concerns.)

1. A feminist therapist would likely consider which intervention most essential? A. a lifestyle analysis B. a functional assessment of a specific problem area. C. social action D. maintaining an anonymous role as a therapist

C ( Whereas non-sexist therapy is more concerned with personal responsibility and personal change, feminist therapy places equal or greater emphasis on the sociopolitical contributions to pathology and the need for social change. The feminist approach to psychotherapy rests on the assumption that social roles and socialization are important determinants of behavior and that psychological conflicts are alternative roles and options. A primary goal of feminist therapy is empowerment or helping women become more self-defining and self-determining. One of the main characteristics of feminist therapy is its emphasis on an egalitarian relationship between the client and therapist. Feminist therapists consider self-disclosure with clients as a way of supporting an egalitarian relationship. Therapist self-disclosure may be used to encourage client participation. Feminist therapy discourages client passivity and feminist therapists typically discourage special bonding as it may encourage client dependence on the therapist. )

6. The term "group polarization" refers to the tendency of groups to make decisions that are A. more risky than those that might be made by individual members. B. more conservative than those that might be made by individual members. C. more risky or more conservative than those that might be made by individual members. D. more illogical than those that might be made by individual members.

C (A group's decisions tend to be more extreme -in one direction or the other- than those that would be made by individuals in the group acting alone. This phenomenon is referred to as group polarization. One explanation for group polarization is that group members are more willing to support extreme decisions because, as group members, they won't have to take as much personal responsibility for their decisions as they would if they were acting alone.)

48. Psychoanalytic theory posits that which of the following is the principle influence at around four months of age? A. reality B. dynamic C. pleasure D. genetic

C (According to Freud, the only personality structure that has developed at birth is the id. The id operates according to the pleasure principle because it looks for ways of gratifying its needs immediately to avoid tension. Answer "A is associated with the ego, which doesn't develop until about six months of age. Freud suggested that the ego operates on the basis of the reality principle because it postpones satisfaction of the id's instincts until there is a suitable object available in reality. The dynamic principle -response "B- refers to Freud's interest in understanding people in terms of the dynamic interactions -conflicts- that occur between the id, ego, and superego, and the genetic principle -response "D- refers to his emphasis on the importance of a person's experiences during childhood.)

37. According to Beck, a depressed man is most likely to believe that: A. the world is unfair, his future is hopeless, but he is a good person B. he is worthless, his future is hopeless, but the world is just C. he is worthless, his future is hopeless, and the world is unfair D. he is worthless, the world is unfair, but his future is hopeful

C (Depressed people tend to distort their perceptions and interpret events from a negative perspective. Beck referred to the "cognitive triaD which consists of negative thoughts about the self, future, and the world. Choice C best represents the cognitive triad.)

53. When trying to prove causation, a researcher mismatches levels of data and tries to apply statistics at one level to infer to data of another level. This is referred to as: A. tautology B. teleology C. ecological fallacy D. latent coding

C (Ecological fallacy is a logic error that occurs when trying to prove causation, levels of data are mismatched and statistics are applied at one level to infer to data of another level. Tautology -a.- is a logic error based on circular reasoning, meaning that something is true by definition or the dependent variable is simply a restatement of the independent variable. Teleology -b.- is a logic error which explains a phenomenon by saying that it was some spirit or higher power that causes the relationship. Latent coding -d.- occurs when a researcher reads into the meaning of the content he/she is analyzing to get data rather than simply taking it at face value. This is in contrast to manifest coding which occurs in content analysis when coding content is based on the face-value rather than looking into the meaning.)

76. Decreased amounts of GABA are most associated with: A. depression B. eating disorders C. Huntington's Disease D. Raynaud's Disease

C (Gamma Amino Butyric Acid -GABA- is the most common inhibitory neurotransmitter in the brain. Low levels of GABA have been linked to several disorders, including Huntington's Disease, Parkinson's Disease, and anxiety disorders.)

98. A child receives a Performance IQ score that is higher than her Verbal IQ score on the WISC-III. When interpreting this difference, it is important to keep in mind that: A. the difference is not significant unless it is 20 points or more. B. the difference may be significant if it is 12 points or more. C. the difference may be significant if it is 12 points or more and the scatter on the Performance subtests is less than 9 points and the scatter on the Verbal subtests is less than 7 points. D. the difference may be significant if it is 18 points or more and the scatter on the Performance subtests is 9 points or more and the scatter on the Verbal subtests is 7 points or more.

C (In general, a 12 point difference between Verbal IQ and Performance IQ on the WISC-III is considered significant. For example, a Performance IQ that is 12 or more points higher than the Verbal IQ might indicate autism, a learning disability, or delinquency. However, if there is also wide scatter of the Verbal and/or Performance subtest scores, the difference between Verbal and Performance IQ may be meaningless.)

84. Indifference and apathy are the likely consequences of lesions in the: A. thalamus, hippocampus, or prefrontal cortex. B. dominant temporal lobe. C. right hemisphere of the cerebral cortex. D. medial hypothalamus.

C (Indifference and apathy are the likely consequences of lesions in certain areas in the right hemisphere of the cerebral cortex. Lesions or damage to the thalamus, hippocampus or prefrontal cortex -a.- have been linked to memory loss. Damage to the dominant temporal lobe -b.-, including the Wernicke's area, causes receptive aphasia. The hypothalamus is associated with regulating the release of hormones from the pituitary and other endocrine glands, maintaining the body's homeostasis, and mediating aggressive responses. In particular, lesions in the medial hypothalamus -d.- have been associated with outbursts of rage and aggressive behavior -See: Swaab, D.F. -2003-. The human hypothalamus: Basic and human aspects, Elsevier Health Sciences, Amsterdam.)

71. A personnel director devises four measures to be used as selection techniques for police officers. She plans on using multiple regression to combine the scores on the four measures in order to predict an applicant's score on a measure of job performance. You suggest that multiple regression is not the appropriate method in this situation because: A. multiple regression is used only when the criterion includes three or more orthogonal categories. B. multiple regression is used only when there are two or more criteria that are each measured on a continuous scale. C. the characteristics assessed by the four measures are noncompensatory; i.e., a low score on one measure cannot be compensated for by a high score on another measure. D. the characteristics assessed by the four measures are compensatory; i.e., a low score on one measure can be compensated for by a high score on another measure.

C (It isn't clear from the question what would be wrong with multiple regression in this situation. However, by process of elimination, only response C is correct. Responses A and B aren't true about multiple regression, and response D is incorrect because multiple regression is compensatory and would be appropriate if a high score on one measure could compensate for a low score on another measure. Multiple regression isn't the appropriate technique when the characteristics measured by the different predictors are noncompensatory.)

51. Which of the following statements regarding the need to obtain informed consent before releasing client information is most true? A. A psychologist should always obtain informed consent from the client before releasing information. B. Before consulting with a colleague about a case, a psychologist must always obtain informed consent from the client. C. A psychologist must obtain informed consent from a client some of the time before releasing information. D. If a psychologist wants to release information about a case, he or she may do so without obtaining informed consent.

C (Ordinarily, you need to obtain informed consent from a client before releasing information you've obtained from him or her. However, this is not always the case; for instance, when you reasonably suspect child abuse, you are legally obligated to file a report with the appropriate state agency -- with or without the client's consent. As for consulting situations -choice B-, informed consent from the client is not necessary as long as you adequately disguise the client's identity.)

29. When Total Quality Management -TQM- fails, it is often because A. there are too few managers. B. there is too much attention to customer demands. C. the employees are not given sufficient responsibility. D. the rewards are not distributed fairly.

C (TQM is an organizational philosophy that focuses on maximizing customer service and satisfaction. An important characteristic of TQM is its involvement of employees in all aspects of decision-making, and failures are often due to management's unwillingness to do this.)

69. The main principle behind the harm reduction approach is: A. prevention is more cost effective than treatment. B. inpatient treatment for substance addicted individuals reduces dangerousness. C. helping individuals not ready to give up substances lead safer lives. D. working towards zero use of substances.

C (The basis of the harm reduction approach is the pragmatic recognition that active substance users must be met "where they are" in terms of their needs and personal goals in treatment. Consistent with a strengths perspective and inspired by cognitive-behavioral therapy, personality theory, traditional addiction treatment, motivational interviewing, relapse prevention, and public health, the emphasis is on reducing the harm of substance abuse and progress over pathology. An alternative to the disease approach, harm reduction approaches embrace the full range of harm-reducing goals including, but not limited to, abstinence and small incremental positive changes are viewed as steps in the right direction. Marlatt -1998- called the philosophy of harm reduction "compassionate pragmatism". -See: Marlatt, G.A. and Tapert., S.F. -1998- Harm Reduction: Pragmatic Strategies for Managing High Risk Behaviors. NY: Guilford Press.)

25. The first step a parent takes in teaching a child to feed the family cat is showing her how to put the cat food into the dish. After mastering that task, she is taught to open a can of cat food and then put it into the dish. The child is then taught to open the pantry door, take out a can of cat food, open it, and put it in the dish. This is an example of: A. stimulus control training B. forward chaining C. backward chaining D. sequential training

C (The child has learned a complex behavior by learning the individual responses in the "behavior chain." The procedure in this question is best described as backward chaining: the parent began by teaching the last behavior and then worked backward from there.)

13. Compared to the Stanford-Binet, the WAIS-III tends to: A. underestimate the IQ scores of higher functioning individuals. B. underestimate the IQ scores of lower functioning individuals. C. overestimate the IQ scores of lower functioning individuals and underestimate the IQ scores of higher functioning individuals. D. underestimate the IQ scores of lower functioning individuals and overestimate the IQ scores of higher functioning individuals.

C (The easiest way to deal with this question and other questions like it is to remember that for the extremes of the IQ continuum, the Stanford-Binet is a better measure to use. It will more accurately reflect the person's functioning at either the very top or very bottom of the range.)

20. According to Atkinson, Sue, and Sue, the ideal outcome of racial/cultural identity development is A. full acceptance of one's own culture. B. giving individual identity priority over cultural identity. C. recognizing that all cultures have acceptable and unacceptable aspects. D. recognizing the impact of racial/cultural oppression on one's mental health.

C (The final stage in Atkinson et al.'s racial/cultural identity development model is referred to as the "integrative awareness stage." Individuals in this stage recognize that all cultures have both positive and negative aspects and decide which aspects they want to accept or reject.)

78. You are working with a couple in marital therapy and are conducting the initial interview. You realize that although the husband doesn't remember you, the two of you once dated. You should A. speak to the husband alone, explain the situation and ask him if he feels comfortable with proceeding. B. see the wife in individual therapy only. C. refer the couple to another therapist. D. let the couple know the situation and then proceed with therapy.

C (This question is fairly easy to answer once you remember you are not obliged to provide services for all your referrals. This is an initial interview and you have a situation that involves a multiple relationship. You need to refer this couple to one of your competent colleagues.)

92. A psychologist serves as the head of a social service agency. The psychologist likes the job because, in addition to allowing her to work with clients in a clinical setting, it provides her with the opportunity for raises and promotions. On the Strong Interest Inventory, the psychologist would likely receive high scores for which of the following personality types? A. investigative and conventional B. realistic and artistic C. social and enterprising D. investigative and enterprising

C (This questions is about Holland's six personality types -- realistic, investigative, artistic, social, enterprising, and conventional. One's results on the Strong Interest Inventory include a 3-letter personality type that indicates, in order, which three types best describe the individual. Individuals in the mental health professions typically obtain a 3-letter code of social, enterprising, and artistic -SEA-; if you knew this, you knew enough to answer the question. If you didn't, the question helped you out by telling you that the psychologist likes to work hands-on with clients -which would be characteristic of the social type- and that she is interested in promotions and pay raises -which would be characteristic of the enterprising type-.)

91. Which of the following is true of transformational leaders? A. the goals of the organization are redefined to reflect the self-interest of followers B. believe that subordinates are persuaded when the organization's goals are compatible with their self-interests C. motivate by encouraging subordinates to transcend self-interest for the greater good of the organization D. assent to the self-interest of their followers

C (Transformational leaders are change-oriented. They encourage and inspire subordinate acceptance and support for organizational change that is in the best interests of the organization. In contrast, transactional leaders motivate through appealing to the followers' self-interests and focus more on stability than change. )

74. Which of the following models of leadership provides a "decision tree" to help a leader determine whether an autocratic, consultative, or consensual decision-making approach is best given the nature of the work situation? A. Fiedler's contingency model B. Hersey and Blanchard's situational model C. Vroom and Yetton's normative model D. House's path-goal model

C (You may have been able to answer this one through the process of elimination if you knew that the models described by Fiedler, Hersey and Blanchard, and House don't include a decision-tree, which is provided by the Vroom and Yetton model.)

60. Which of the following statements is true regarding the incidence of Major Depressive Disorder? A. The rate for prepubescent girls is twice that of prepubescent boys B. The rate for adolescent females is half that of adult females C. The rate for adolescent and adult females is half that of adolescent and adult males D. The rate for adolescent and adult females is twice that of adolescent and adult males

D (According to the DSM-IV-TR, Major Depressive Disorder occurs at twice the rate for adolescent and adult females as compared to adolescent and adult males. The rates for prepubertal boys and girls are equal.)

40. A forensic psychologist is asked to review and make conclusions regarding a defendant's psychological evaluation that was conducted 4 years ago following a similar crime that the defendant is now accused of. The psychologist should: A. refuse the request B. comply with the request but interpret the previous findings with caution C. reevaluate the person and disregard the previous findings D. reevaluate the person and interpret the previous findings with caution

D (According to the Forensic Specialty Guidelines, "Forensic psychologists avoid giving written or oral evidence about the psychological characteristics of particular individuals when they have not had an opportunity to conduct an examination of the individual..." And, according to Ethical Standard 9.08, psychologists also do not base their decisions on outdated results. Thus, the psychologist in this case should make a reasonable effort to reevaluate the person. However, it would probably be inappropriate to completely disregard the previous findings. The best approach would be to reevaluate the person and interpret the previous findings with caution.)

90. A psychologist determines clients' fees based on their current income. This "sliding scale" practice is: A. explicitly prohibited in the Ethics Code B. explicitly recommended in the Ethics Code C. unacceptable but not explicitly mentioned in the Ethics Code. D. acceptable but not explicitly mentioned in the Ethics Code.

D (As long as they are fair and serve the best interests of the client, sliding scale fees are generally considered acceptable. The Ethics Code does not explicitly address sliding scale fees.)

5. Thinning refers to the process of: A. switching from a fixed interval to a fixed ratio schedule B. switching from a variable interval to a variable ratio schedule C. switching from an intermittent to a continuous schedule D. switching from a continuous to an intermittent schedule

D (Continuous schedules, or reinforcing every response, are associated with quick learning, satiation and extinction. The process of thinning, or switching from a continuous to an intermittent schedule, is used to increase the resistance to extinction once a behavior is established.)

86. Individuals with Major Depressive Disorders who experience abnormalities in the sleep cycle may have any of the following sleep disturbances EXCEPT: A. early morning awakening B. sleep continuity decrease C. REM latency decrease D. slow-wave sleep increase

D (Depression is associated with decreased slow-wave or non-REM sleep as well as, early morning waking -a.-, decreased sleep continuity -b.- and earlier onset of REM sleep or decreased REM latency -c.-. )

99. Recent research on HIV infection has examined the role of psychosocial risk factors on disease progression and prognosis. The results of studies involving infected adults has most consistently found: A. lower intellectual functioning and younger age are related to a more rapid progression B. higher intellectual functioning and older age are related to a more rapid progression C. somatic symptoms of depression and younger age are related to a more rapid progression D. somatic symptoms of depression and older age are related to a more rapid progression

D (Findings of recent studies indicate that the factors of intellectual functioning, age and somatic symptoms of depression are significant predictors of HIV progression and prognosis. Specifically, lower IQ, older age and the presence of somatic symptoms of depression are associated with a more rapid progression from HIV infection to AIDS, HIV-related dementia, and death. -See: Farinpour, R., et al., Psychosocial risk factors of HIV morbidity and mortality: Findings from the Multicenter Aids Cohort Study -MACS-, Journal of Clinical and Experimental Neuropsychology, 2003, 25-5-, 654-670.)

62. Jose scored 75 on his final exam. The test scores were normally distributed, with a mean of 60 and a standard deviation of 15. Jose's score would be in which of the following percentile ranges? A. 35-49 B. 50-64 C. 65-79 D. 80-95

D (In a normal distribution, 1.0 is 34 percentile points above the mean of 50. Jose's standard score is -75-60-/15 or 1.0, putting his score at the 84th percentile. )

64. Right-left confusion is a characteristic of damage to the: A. temporal lobe B. frontal lobe C. occipital lobe D. parietal lobe

D (Left-right disorientation is typically caused by damage in the parietal lobe, specifically the left angular gyrus located between the parietal and temporal lobes. It is also one of the symptoms of Gerstmann's syndrome.)

89. Which of the following factors are considered most important for competent multicultural counseling? A. commitment, sensitivity, and technique B. sensitivity, instillation of hope, and acceptance C. acceptance, awareness, and flexibility D. awareness, knowledge, and skills

D (Multicultural competence is most often attributed to one's level of awareness, knowledge, and skills. "Awareness" refers to awareness of one's own beliefs, values, and stereotypes. "Knowledge" refers to knowledge of the worldviews of clients with different cultural backgrounds. And "skills" refers to the skills that are most appropriate for clients with different cultural backgrounds -D. W. Sue, Multidimensional facets of cultural competence. Counseling Psychologist, 2001, 29-6-, 790-821-.)

97. In comparison studies of younger and older adults, it has been found that depression in older adults is least likely to result in A. difficulties with memory problems. B. anxiety feelings. C. feelings of hopelessness. D. expressed sadness.

D (Older adults are less likely than younger adults to express feelings of depression or sadness. They are more willing to express feelings of hopelessness -answer C- and anxiety -answer B-. They are also more apt to have memory problems -answer A-. -APA Working Group on the Older Adult, What practitioners should know about working with older clients, Professional Psychology: Research and Practice, 1998, 29-5-, 413-427-.)

55. Which of the following techniques will exacerbate of chronic pain rather than reduce it? A. Religious coping B. Cognitive therapy C. Active coping D. Passive coping

D (Passive pain-coping strategies, a category described in Brown and Nicassio's -1987- dichotomy of active versus passive coping strategies, are associated with worse pain and adjustment among chronic pain patients. Passive coping strategies are those that involve giving responsibility for pain management to an outside source or allowing other areas of life to be adversely affected by pain. They may also serve as psychological enforcers of pain. Examples of passive coping strategies are focusing on where the pain is and how much it hurts, restricting or cancelling social activities, or thoughts such as " There's nothing I can do to lessen this pain," or "I wish my doctor would prescribe me better pain medication." In contrast, active coping strategies entail the patient taking responsibility for pain management including attempts to control the pain or to function in spite of it. Examples of active coping strategies include engaging in physical therapy or exercise, staying busy/active, relaxation techniques, clearing distracting thoughts and attention from the pain. Cognitive -b.- and cognitive behavior therapy has been shown to be effective in reducing the experience of pain and improving positive behavior expression, appraisal and coping in individuals with chronic pain. -See: Morley S, Eccleston C, Williams A. -1999- Systematic review and meta-analysis of randomised controlled trials of cognitive behaviour therapy and behaviour therapy for chronic pain in adults, excluding headache. Pain, 80, 1-13.- Evidence supports a relationship between positive health outcomes and the use of religious coping to manage pain. Pargament -1990- identified three possible interactions between religion and coping: religion can influence the parts of the coping process -appraisal, coping activities, results, assistance, and motivation-; contribute to the coping process by influencing perception -attribution of meaning and the feeling of control- and preventing certain events from happening -through a beneficiary life style-; and it can be the resultant of the coping process through religious attributes. -See: Pargament, K.I. -1997- The psychology of religion and coping, Guilford, New York.)

65. Research comparing gifted and average children indicates that gifted children tend to be superior in terms of: A. response speed B. parallel processing C. divergent thinking D. metacognition

D (Research on giftedness has focused on the role of metacognition and has consistently shown that gifted children are more aware of their cognitive processes and better at choosing and applying cognitive strategies and evaluating the effectiveness of their choice. Response accuracy is more characteristic of gifted children than response speed -a.-. Parallel processing -b.- is a term that has several meanings. In the context of attention and memory, it refers to the flow of information from the source to the recipient via more than one route. It has not been linked to giftedness. Divergent thinking -c.- is found in both gifted and average children and is associated with creativity.)

18. According to the catecholamine hypothesis, depression is due to a deficiency in: A. dopamine B. gamma-aminobutyric acid C. acetylcholine D. norepinephrine

D (Research with drugs led to the formulation of the "catecholamine hypothesis" about the cause of mood disorders. First presented in 1965, Schildkraut et al.'s theory of depression emphasized the association of depression with a decrease in the levels of brain catecholamines dopamine -DA- and norepinephrine -NE-, especially norepinephrine. This hypothesis in its simplest form, stated that depression was due to a deficiency of norepinephrine, one of the major catecholamine systems in the brain. This early hypothesis is based on evidence from three groups of drugs: reserpine, monoamine oxidase -MAO- inhibitors, and tricyclics. Reserpine depletes the brain of dopamine and norepinephrine and is used primarily to treat hypertension. It is also used for severe agitation. MAO inhibiters and tricyclics, although they work in somewhat different ways, are used to treat depression and tend to increase the amount of norepinephrine available in the central nervous system. A variety of approaches, especially in the United States, tested this hypothesis and the results are inconclusive. Nonetheless, this theory served as the "standarD theory for depression in the United States until the 1980s. -See: McNeal, E. T., and Cimbolic, P. -1986-. Antidepressants and Biochemical Theories of Depression. Psychological Bulletin, 99-3-, 361-374.)

94. Research by Sue and his colleagues -1991- suggests that which of the following clients is most likely to return for a second session of psychotherapy? A. an African-American client B. a Latino-American client C. an Anglo-American client D. an Asian-American client

D (Studies on therapy dropout rates have produced inconsistent results. This question is asking about a particular study, however -Sue et al., 1991-, which found that African-Americans have the highest dropout rates, while Asian-Americans have the lowest dropout rates.)

54. A self-report inventory that assesses general psychiatric symptoms on a Likert-type scale is the: A. MCMI-III B. Rorschach C. MMPI-II D. SCL-90

D (The Symptom Checklist 90 -SCL-90- is a self-report inventory assessing general psychiatric symptoms of anxiety, depression, somatization, obsessive-compulsiveness, and hostility. Clients respond to the stimulus term, such as "feel nervous inside" on a 5-point Likert type scale from "not at all" which gives a score of 0, to "extremely" which gives a score of 4.)

95. You have conducted a study assessing the relationship between salary and job performance, and you find a significant correlation between these two variables. Your assistant tells you that the data fail to take into account a $25.00 cost of living raise which every employee received. You should: A. decide that the raise invalidated the research. B. reanalyze the data after the raises have been added to the current salary. C. not worry about small details; the actual amount is too small to make a significant difference. D. assume the correlation will not be affected.

D (The basic point being tested here is that if you add a constant to each score -- in either or both data sets -- the relationship between the two variables won't be affected. In other words, adding a constant to every score does not affect the correlation coefficient. The same is true of multiplying or dividing all scores by a constant, or subtracting a constant from every score.)

26. Kohlberg would agree with all of the following except: A. moral development is an outgrowth of cognitive development. B. each stage of development represents an organized whole. C. stage 5 and 6 are not reached by most people. D. moral development stages have an inherent male bias.

D (The first three answers are descriptions of Kohlberg's theory. Answer D is Carol Gilligan's criticism of Kohlberg. Gilligan thought males are likely to refer to principles of justice and fairness when making decisions, while females are more likely to refer to interpersonal connectedness and care. Research has not generally supported Gilligan's hypothesis.)

81. Introducing a goal that requires cooperation between conflicting intergroups results in: A. increased hositility, increased competition B. increased hostility, reduced competition C. reduced hositility, increased competition D. reduced hostility, reduced competition

D (The introduction of a superordinate goal, one that requires cooperation to accomplish, was found to be the most effective way of reducing or alleviating intergroup hostility and competition in Sherif et al.'s "Robber's Cave" study.)

47. You have been attempting to collect payment from a former patient of yours who did not pay for his final two months of therapy. One day, you get a letter from a state hospital where that former patient is now currently a patient. The letter includes a request for a copy of your records of the patient's treatment along with a signed release of information. In this situation, you should A. refuse to supply any information to the hospital until the bill is paid. B. send a summary of the records to the hospital along with a copy of a bill for the unpaid therapy fees. C. go to the hospital and talk to the patient about the situation. D. provide the information requested by the hospital.

D (The issue here is that you can't withhold records under your control that are "requested and needed for a client's/patient's emergency treatment solely because payment has not been received -Ethical Standard 6.03-. Thus, you must provide the requested information in this instance.)

35. If you hear arguments against your opinion, followed by arguments against the opposing opinion, what is likely to happen? A. You will become very confused. B. You will become more dogmatic in your original opinion. C. You will change your opinion. D. Your resistance to future opposing arguments will be increased.

D (This question is indirectly referencing McGuire's inoculation theory, which proposes that a particular attitude or belief can be strengthened by exposing someone to the opposing belief -- especially when the opposing argument is weak or the person is supplied with counter-arguments against the opposing belief. Note that this is analogous to medical inoculation, which involves injection of a weak form of a germ so the body can build up defenses against that germ.)

57. Which of the following statements is not contained in APA's Ethical Principles of Psychologists and Code of Conduct? A. "the purely private conduct of psychologists ... is not within the purview of the Ethics Code. B. "This Ethics Code ... has as its goals the welfare and protection of the individuals and groups with whom psychologists work ..." C. "The fact that a given conduct is not specifically addressed by an Ethical Standard does not mean that it is necessarily either ethical or unethical." D. "A violation of the Ethics Code usually means ... that a psychologist will be legally liable in a court action or face other legal consequences."

D (To answer this question, you didn't need to have had the Ethics Code memorized. All you needed was a good understanding of what the Code is all about. Choice D is not part of the Ethics Code, since ethical and legal standards, even though they sometimes overlap, are distinct, with the Ethics Code often holding psychologists to more stringent standards. In fact, the Code states that "whether or not a psychologist has violated the Ethics Code does not by itself determine whether he or she is legally liable in a court action.")


Conjuntos de estudio relacionados

rad physics homework questions quiz

View Set

AP European History IDs of Chapters 12-30

View Set

CIT 372 Review Questions quizzes

View Set

5 Axilla and arm anatomy - trebloc

View Set

Environmental Science Ch. 19: Waste

View Set